Você está na página 1de 91

MA 21 – Resolução de Problemas

Professor: Sandro Rodrigues Mazorche

Grupo: 2

Márcio Eduardo Primo

Brasílio Freitas

Paulo Geovane

Paulo Fernandes

Marcelo de Moura Costa


Capítulo 1 – Soluções dos Exercícios

1. Uma sacola contém meias cujas cores são branca, preta, amarela e azul. Sem olhar para a sacola,
qual é a quantidade mínima de meias que precisamos retirar da mesma para garantir pelo menos um
par de meias da mesma cor?

Precisamos retirar no mínimo 5 meias. Como o número de meias será maior que o número de
cores teremos certeza absoluta que formaremos um par de mesma cor.

2. O pai do padre é filho único de meu pai. O que eu sou do padre?

Como o filho único do meu pai sou eu, então, sou o pai do padre.

3. Numa mesa há 5 cartas:

Q T 3 4 6

Cada carta tem de um lado um número natural e do outro lado uma letra. João afirma: "Qualquer carta
que tenha uma vogal tem um número par do outro lado". Pedro provou que João mente virando
somente uma das cartas. Qual das 5 cartas foi a que Pedro virou?

Pedro virou a carta 3, mostrando que havia uma vogal escrita no verso.

4. A polícia prende 4 homens, um dos quais é culpado de um furto.

Eles fazem as seguintes declarações:

_ Arnaldo: Bernaldo é o culpável.

_ Bernaldo: Cernaldo é o culpável.

_ Dernaldo: eu não sou culpável.

_ Cernaldo: Bernaldo mente ao dizer que eu sou culpável.

Só uma destas declarações é a verdadeira, quem é culpável pelo furto?

Testando cada uma das afirmações, temos que ter todas as outras falsas.

Suponha então:

(i) Arnaldo fala a verdade. Então: Bernaldo é o culpável. Nesse caso:

1. Benaldo mente. (Falso)

2. Dernaldo fala a verdade. Isso é suficiente para descartar que Arnaldo fale a verdade.
(ii) Bernaldo fala a verdade. Então: Cernaldo é o culpável. Nesse caso:

1. Arnaldo mente. (Falso)

2. Dernaldo fala a verdade. Descartamos ser Bernaldo que falou a verdade.

(iii) Dernaldo fala a verdade. Então: ele não é culpável.

1. Arnaldo pode estar falando verdade ou não

2. Bernaldo pode estar falando verdade ou não

3. Cernaldo pode estar falando verdade ou não. Porém, se a afirmação de Cernaldo for verdadeira a
de Bernaldo é necessariamente falsa e vice versa. Ou seja, uma das duas declarações, de Cernaldo
ou Bernaldo também será verdadeira, o que descara a possibilidade de que Dernaldo seja o único que
fala a verdade.

(iv) Cernaldo fala a verdade. Só restou essa afirmação como possivelmente verdadeira. Nesse caso:
Bernaldo mente, e Cernaldo não é culpável.

1. Arnaldo pode estar falando a verdade ou não

2. Bernaldo mente (hipótese de que Cernaldo fale a verdade)

3. Dernaldo pode estar falando a verdade ou não

Concluímos que apenas Cernaldo pode ter falado a verdade, restando que todos outros mentem.

Quem seria o culpável?

Não é Bernaldo e nem Cernaldo, pois Arnaldo e Bernaldo falam mentira.

Dernaldo também mente, então necessariamente ele é o culpável.

5. Numa cidade existe uma pessoa X que sempre mente terças, quintas e sábados e é completamente
sincera o resto dos dias da semana. Felipe chega um certo dia na cidade e mantém o seguinte diálogo
com a pessoa X:

_ Felipe: Que dia é hoje?

_ X: Sábado.

_ Felipe: Que dia será amanhã?

_ X: Quarta-feira.

Em qual dia da semana foi mantido este diálogo?

X necessariamente está no dia de mentir, pois, não é possível hoje ser sábado e amanhã ser
quarta-feira, portanto, pode ser terça, quinta ou sábado. Se fosse terça-feira, a afirmação “Que dia será
amanhã?” seria verdadeira. Se fosse Sábado a afirmação “Que dia é hoje?” seria verdadeira. Portanto,
o diálogo só pode ter acontecido na quinta-feira.
6. Divida o relógio de parede abaixo em 6 partes iguais de forma tal que a soma das horas que ficam
em cada parte seja a mesma.

A soma dos valores: 1 + 2 + ... + 11 + 12 = 78. Dividindo por 6 temos


quociente 13. Portanto devemos ter 6 partes de soma 13. Teremos os pares
(12, 1); (11, 2); (10, 3); (9, 4); (8, 5) e (6, 7).

Para realizar a divisão em partes iguais deve ser procurada uma


disposição geométrica que satisfaça essa condição.

7. João adora Gabriela, que é uma aluna excelente em Matemática. João armou um plano para dar um
beijo nela, e descobriu que poderá fazer isso apenas dizendo uma frase. Que frase é essa?

"Se gostas de Matemática deves me dar um beijo, mas se não gostas eu a beijarei".

8. No plano se colocam 187 rodas dentadas do mesmo diâmetro, enumeradas de 1 até 187. A roda 1 é
acoplada com a roda 2, a 2 com a 3, : : : , a 186 com a 187 e esta última com a roda 1. Pode tal
sistema girar?

As rodas ímpares giram em um sentido e as rodas pares no sentido contrário. Como a roda 187
será acoplada na roda 1 (ambas ímpares) não será possível que o sistema gire.

9. Um canal, em forma quadrada, de 4 metros de largura rodeia um castelo. A ponte do castelo está
fechada e um intruso quer entrar no castelo usando duas pranchas de 3,5 metros de comprimento.
Será que o intruso consegue?

Sendo: AK = AL temos que AM = 1,75 m

Como AB = 4 2 m

MB = 4 2 – 1,75  3,91 m

Como a outra prancha tem 3,5 m não é possível atravessar o fosso.


10. Os números 1, 2, 3, . . . ,99 são escritos no quadro-negro e é permitido realizar a seguinte
operação: apagar dois deles e substituí-los pela diferença do maior com o menor. Fazemos esta
operação sucessivamente até restar apenas um último número no quadro. Pode o último número que
restou ser o zero?

Na sequência 1, 2, ... , 99 excluí-se o 1 e subtraí-se os números consecutivos dois a dois,


obteremos 50 números 1 (25 pares) que subtraídos dois a dois, resulta zero.

11. Alguém elege dois números, não necessariamente distintos, no conjunto de números naturais 2, ...,
20. O valor da soma destes números é dado somente a Adriano (A) e o valor do produto dos números
é dado unicamente a Karla (K).

_ Pelo telefone A diz a K: _Não é possível que descubras minha soma.

_ Uma hora mais tarde, K diz a A: _Ah! Sabendo disso, já sei quanto vale tua soma!

_ Mais tarde A chama outra vez K e lhe informa: _Poxa, agora eu também conheço teu produto!

Quais números foram eleitos?

Não conseguimos resolver!

Na revista EUREKA! no 23 de 2006 foi publicada uma resolução desse problema pelo Professor
Cássio Neri do Instituto de Matemática da Universidade Federal do Rio de Janeiro.

O artigo tem por título "O Problema Impossível" e cita que a forma original foi proposta por
Hans Freudenthal e popularizada por Martin Gardner.

Para solucionar, o Professor Cássio usa um algoritmo computacional em linguagem C


(disponível em www.labma.ufrj.br/~cassio/f-impossivel.html) e um método não computacional.

Após as considerações, em ambos os casos, ele determina que os números eleitos são 4 e 13.

12. É possível cobrir um tabuleiro de xadrez com 31 dominós onde removemos as casas dos vértices
superior esquerdo e inferior direito?

Não pois as casas retiradas estão na mesma diagonal, são casas de mesma cor.

13. Num saco encontram-se 64 moedas leves e 64 moedas pesadas. É possível separar duas moedas
de pesos diferentes com 7 pesagens?

Inicialmente dividimos as moedas em dois grupos de 64 moedas. Colocamos cada grupo nos
pratos da balança.
Na segunda pesagem escolha as 64 moedas do prato mais leve, caso fiquem desequilibrados,
ou qualquer grupo se ficaram em equilíbrio e separe em dois grupos de 32 moedas em cada prato da
balança.

Para realizar a terceira pesagem, proceda como na segunda, separando agora 16 moedas em
cada prato da balança.

Na quarta pesagem escolha um dos grupos de 16 e divida-o em dois grupos de 8 moedas para
cada prato da balança.

Chegando na quinta pesagem divida as oito moedas obtidas na pesagem anterior e divida em
dois grupos de quatro moedas para cada prato.

Agora na sexta pesagem divida um dos grupos de 4 moedas em dois grupos de 2 moedas para
cada prato.

Na sétima, e última pesagem, escolha um grupo de 2 moedas da sexta pesagem e coloque


uma moeda em cada prato. Se houver desequilíbrio as moedas são as que estão nos pratos. Se
houver equilíbrio as moedas de pesos diferentes são as outras duas separadas anteriormente.

14. Quantas vezes precisamos dobrar um papel de 1mm de espessura para que a altura da pilha
chegue da Terra à Lua?

Cada dobra efetuada nos fornecerá uma nova altura que corresponderá ao dobro da altura
anterior. A sequência formada será uma PG de razão 2. Portanto teremos que encontrar um termo na
PG que seja igual à distância da Terra à Lua.

Seja X a distância, em milímetros, da Terra à Lua teremos então:

X  1.2 n1  X  2n : 2  2. X  2n  log (2 . X )  log 2 n 

log 2  log X log X


log 2  log X  n. log 2  n  n  1
log 2 log 2

15. Descubra o erro da prova da afirmação abaixo:

Afirmação: Três é igual a dois.

Seja x um número diferente de zero. Temos que:

3x – 3x = 2x – 2x

Colocando x – x em evidência, temos que:

3(x – x) = 2(x – x).

Cancelando x – x em ambos os lados, obtemos que 3 = 2.

O erro está no cancelamento de x – x. Nessa passagem é efetuada uma divisão por zero.
Capítulo 2 – Soluções dos Exercícios

1. Observe as multiplicações a seguir:

(a) 12.345.679 x 18 = 222.222.222

(b) 12.345.679 x 27 = 333.333.333

(c) 12.345.679 x 54 = 666.666.666

Para obter 999.999.999 devemos multiplicar 12.345.679 por quanto?

Solução:

O algarismo que desejo repetir resulta da multiplicação do próprio por 9, ou seja:

12.345.679 x 9.n = nnn.nnn.nnn (com 0 < n < 10, sendo n um número natural)

Como desejo repetir o algarismo 9, temos que multiplicar por 81.

2. Outro dia ganhei 250 reais, incluindo o pagamento de horas.extras. O salário (sem horas extras)
excede em 200 reais o que recebi pelas horas extras. Qual é o meu salário sem horas extras?

Solução:

Denominando S = salário e HE = hora extra, teremos:


(1) S + HE = 250
(2) S = HE + 200
Substituindo a relação (2) em (1), teremos:
200 + HE + HE = 250
HE = 25
S = 225

O meu salário sem horas extras é de R$225,00.

3. Uma torneira A enche sozinha um tanque em 10 h, uma torneira B enche o mesmo tanque sozinha
em 15 h. Em quantas horas as duas torneiras juntas encherão o tanque?

Solução: Temos que:

⇒ ⇒ T = 6 horas

4. O dobro de um número, mais a sua terça parte, mais a sua quarta parte somam 31. Determine o
número.

Solução:
Denominando de x o número a ser descoberto:

24x + 4x + 3x = 372

31x = 372

x = 12
5. Uma certa importância deve ser dividida entre 10 pessoas em partes iguais. Se a partilha fosse feita
somente entre 8 dessas pessoas, cada uma destas receberia R$5.000,00 a mais. Calcule a
importância.

Solução:
Denominando:
P = parte que cada pessoa irá receber;
T = total a ser dividido.

4T + 200.000 = 5T

T = 200.000

A importância é de R$200.000,00.

6. Roberto disse a Valéria: “Pense um número, dobre esse número, some 12 ao resultado, divida o
novo resultado por 2. Quanto deu?” Valéria disse “15” ao Roberto, que imediatamente revelou o
número original que Valéria havia pensado. Calcule esse número.

Solução: Seja x o número pensado por Valéria, Temos que:

1º) Passo: 2x

2º) Passo: 2x + 12

3º) Passo:

4º) Passo: x + 6 = 15

Logo; x = 9

7. Por 2/3 de um lote de peças iguais, um comerciante pagou R$8.000,00 a mais do que pagaria pelos
2/5 do mesmo lote. Qual o preço do lote todo?

Solução:

Denominando de x o valor total do lote, teremos:

10x – 120.000 = 6x

x = 30.000

O valor total do lote é de R$30.000,00

8. Determine um número real a para que as expressões 3a+6/8 e 2a+10/6 sejam iguais.

Solução:
9a +18 = 8a + 40

a = 22

9. Se você multiplicar um número real x por ele mesmo e do resultado subtrair 14, você vai obter o
quíntuplo do número x. Qual é esse número?

Solução:

Temos: { . * +

10. Eu tenho o dobro da idade que tu tinhas quando eu tinha a tua idade. Quando tu tiveres a minha
idade, a soma das nossas idades será de 45 anos. Quais são as nossas idades?

Solução:

Denominando:
Minha idade = y
Sua idade = x
A diferença entre as idades = k
Teremos então:

(1) y = 2.(x – k)
(2) x + k + y + k = 45
Substituindo (1) em (2)
3x = 45
x = 15
Como y = 2x – 2k e y – x = k, teremos
y=k+x
y = k + 15
k + 15 = 30 – 2k
k=5
y = 5 + 15 = 20

Nossas idades são: eu tenho 20 anos e você 15 anos.

11. Um homem gastou tudo o que tinha no bolso em três lojas. Em cada uma gastou 1 real a mais do
que a metade do que tinha ao entrar. Quanto o homem tinha ao entrar na primeira loja?

Solução:
Denominando de x o valor com o qual o homem tenha entrado no local.
Resolvendo o problema de trás para frente, teremos:
3º local: x/2 – 1 = 0, x = 2, logo, ele entrou com R$2,00.
2º local: x/2 – 1 = 2, x = 6, logo, ele entrou com R$6,00.
1º local: x/2 – 1 = 6, x = 14, logo, ele entrou com R$14,00.

Ele entrou na primeira loja com R$14,00.


12. Com os algarismos x, y e z formam-se os números de dois algarismos xy e yx, cuja soma é o
número de três algarismos zxz. Quanto valem x, y e z?

Solução:

Os algarismos são xy e yx, e a soma é zxz. Considerando os algarismos como x, y, z Temos que (10x
+ y) + (10y + x) = 100z + 10x + z e 11x + 11y = 101z + 10x (ou ainda x + 11y = 101z)

O número da forma zxz precisa estar entre 101 e 98 + 89 = 187, ou seja 101 < zxz <181.
Como o número é 11x + 11y = 11(x + y), temos que o número é um múltiplo de 11. Ou seja, as
soluções possíveis são os múltiplos de 11 entre 101 e 181, ou seja: 110, 121, 132, 143, 154, 165, 176.
Dentre esses, apenas o 121 tem o algarismo da centena igual ao da unidade.

Logo z = 1, x = 2. Precisamos encontrar y. Ora, x + 11y = 101z, como já enunciamos.

Fazendo as substituições: 2 + 11y = 101 => 11y = 99 => y = 9

Os algarismos são x = 2, y = 9, z = 1

13. Quantos são os números inteiros de 2 algarismos que são iguais ao dobro do produto de seus
algarismos?

Solução:

Assumindo que os algarismos que compõe o número sejam denominados de x(algarismo das
dezenas) e y(algarismo das unidades), teremos:

10x + y = 2xy

O dobro do produto dos algarismos implica que o número seja par, logo, o algarismo das unidades só
poderá ser: 0,2,4,6 ou 8
Porém podemos excluir y = 0 pois o produto dos algarismos seria zero.
Se y = 2, temos que 10x + 2 = 4x, solução descartada.
Se y = 4, temos que 10x + 4 = 8x, solução descartada.
Se y = 6, temos que 10x + 6 = 12x, então x = 3.
Se y = 8, temos que 10x + 8 = 16x, solução descartada.

O único número que atende o problema é o 36.

14. Obter dois números consecutivos inteiros cuja soma seja igual a 57.

Solução:

Denominando um número de x e seu posterior de x + 1, teremos:

x + (x + 1) = 57
x = 28

Os números que atendem a solução do problema são 28 e 29.


15. Qual é o número que, adicionado ao triplo do seu quadrado, vale 14?

Solução:

Chamando o número de x, temos:

x + 3x2 = 14  x = – 7/3 ou x = 2.

16. O produto de um número positivo pela sua terça parte é igual a 12. Qual é esse número?

Solução:

Denominando o número de x, teremos:

x2 = 36

x =  6, mas como o enunciado diz que o número é positivo, x = 6.

17. Determine dois números consecutivos ímpares cujo produto seja 195.

Solução:

Denominando um número ímpar por 2x + 1, e seu consecutivo por 2x + 3, teremos:

(2x + 1).(2x + 3) = 195


4x2 + 8x – 192 = 0
x' = – 8 ou x’’ = 6, fazendo as devidas substituições teremos:
Ou os números são 13 e 15 ou – 15 e – 13.

18. A diferença entre as idades de dois irmãos é 3 anos e o produto de suas idades é 270. Qual é a
idade de cada um?

Solução:

Chamando as idades de x e y, temos duas equações

y2 + 3y – 270 = 0
P = – 270
S=–3
y = 15 ou y = –18

Como x e y representam idades consideramos apenas o valor positivo. Portanto y = 15 e x = 18. Então
as idades dos irmãos são 15 e 18 anos

19. Calcule as dimensões de um retângulo de 16 cm de perímetro e 15 cm2 de área.

Solução:

Denominando as dimensões de x e y, temos pelo perímetro e pela área que:

(pelo perímetro) 2x + 2y = 16
x+y=8
y=x–8
(pela área) x.y = 15
(substituindo y) x.(x – 8) = 15
x2 – 8x – 15 = 0
x’ = 5 ou x’’ = – 3
Como estamos tratando de dimensões geométricas, usaremos somente o valor positivo, logo:
x=5ey=3

20. A diferença de um número e o seu inverso é 8/3 . Qual é esse número?

Solução:

Denominando o número de x, teremos:

3x2 – 3 = 8x
3x2 – 8x – 3 = 0
x' = 3 ou x’’ = – 1/3

Questão 21: A soma de dois números é 12 e a soma de seus quadrados é 74. Determine os dois
números.

Solução:

Chamando os dois números de x e y temos:

Da primeira equação, temos que:

(x + y)2 = (12)2
x2 + y2 + 2xy = 144
(substituindo pela segunda equação) 74 + 2xy = 144
2xy = 70
xy = 35
x = 35/y
(substituindo na primeira equação)
35 + y2 = 12y
2
y – 12y + 35 = 0
y’ = 5 ou y’’ = 7
Se y = 5, x = 7 ou se y = 7, x = 5

Os números são 7 e 5.
22. Um pai tinha 30 anos quando seu filho nasceu. Se multiplicarmos as idades que possuem hoje,
obtém-se um produto que é igual a três vezes o quadrado da idade do filho. Quais são as suas idades?

Solução:

Denominando de P = idade do pai, e F = idade do filho, temos que:

P – F = 30
PF = 3F2 → P = 3F
(substituindo na primeira equação) 3F – F = 30
F = 15 e P = 45

O pai tem 45 anos enquanto que o filho tem 15 anos.

23. Os elefantes de um zoológico estão de dieta juntos. Num período de 10 dias devem comer uma
quantidade de cenouras igual ao quadrado da quantidade que um coelho come em 30 dias. Em um dia
os elefantes e o coelho comem juntos 1.444 kg de cenoura. Quantos quilos de cenoura os elefantes
comem em 1 dia?

Solução:

Denominando de E = quantidade que os elefantes devem comer em um dia e C = quantidade que um


coelho deve comer em um dia, teremos:

{ ⇒ {
( )

10C = 14440 – 900C2


90C2 + C – 1444 = 0
C’ = 4 C’’< 0 (descartado)
E = 1440 kg

Um elefante come 1440 kg de cenoura por dia.

Questão 24: Sejam α1 e α2 as raízes do polinômio ax2 + bx + c, com a ≠ 0. Calcule as seguintes


expressões, em função de a, b e c:

(a)
(b) √ √
(c) √ √

Solução:

a) ⇒

b) ( ) √ √

√ √ √ √

c) ( √ √ ) √ √ √ √ √ √
√ √ √√ √ √

25. O número – 3 é a raiz da equação x2 – 7x – 2c = 0. Nessas condições, determine o valor do


coeficiente c.

Solução:

Se –3 é raiz, então: p(– 3) = 0


(– 3)2 – 7.( – 3) – 2.c = 0
c = 15

26. Encontre o polinômio p(x) = 2x4 + bx3 + cx2 + dx+ e que satisfaz a equação p(x) = p(1 - x).

Solução:

p(x) = 2x4 + bx3 + cx2 + dx+ e


p(x – 1) = 2(x – 1)4 + b(x – 1)3 + c(x – 1)2 + d(x – 1)+ e
p(x – 1) = 2x + (– 8 – b)x3 + (12 + 3b + c)x2 + (– 8 – 3b – 2c – d)x + (2 + b + c + d + e)
4

Pela identidade de polinômios, temos que: p(x) ≡ (p – 1), comparando os coeficientes de mesmo grau
iremos concluir que:
b = – 4, c = t (t  ℝ), d = 2 – t, e  ℝ

O polinômio procurado será: p(x) = 2x4 – 4x3 + tx2 + (2 – t )x+ e

Questão 27: (OBM) Dois meninos jogam o seguinte jogo. O primeiro escolhe dois números inteiros
diferentes de zero e o segundo monta uma equação do segundo grau usando como coeficientes os
dois números escolhidos pelo primeiro jogador e 1.998, na ordem que quiser (ou seja, se o primeiro
jogador escolhe a e b o segundo jogador pode montar a equação 1.998x2 + ax + b = 0 ou ax2 + 1.998x
+ b = 0 etc.) O primeiro jogador é considerado vencedor se a equação tiver duas raízes racionais
diferentes. Mostre que o primeiro jogador pode ganhar sempre.

Solução:

Inicialmente veja que, se num polinômio p(x) = anxn + … + a1x + a0, tivermos an + … a1 + a0 = 0,
teremos p (1) = 0 e 1 é raiz de p. Dessa forma se o primeiro jogador escolhe b = – (1998 + a), 1 será
raiz da equação do segundo grau que o seu oponente irá montar.
Se um polinômio tem coeficientes inteiros (na verdade vale para coeficientes racionais) e possui uma
raiz irracional do tipo a + b r (r não é quadrado perfeito), então a – b r também é raiz. Ou seja, as
raizes irracionais vêm aos pares. No caso de uma equação de segundo grau, e coeficientes inteiros,
ambas as raizes são irracionais, ou ambas são racionais.
No nosso caso, como 1 já é raiz, a outra raiz será racional. Basta ver então, apenas se 1 não é raiz
multipla (pois queremos que as raizes sejam distintas). Para isso basta escolher a adequadamente.
Se o primeiro jogador escolher os números a = n1998, e b = – (n + 1) 1998, n  2  N, ele ganha. (1
não será raiz múltipla e a equação terá duas raizes racionais distintas.)

Obs. É possivel obter soluções com a + b + 1998  0, por exemplo com

{a, b} = {2040, – 5478} (solução obtida com o auxílio de um computador.)


28. (OBM) Mostre que a equação x2 + y2 + z2 = 3xyz tem infinitas soluções onde x;y;z são números
inteiros.

Solução:

Escolhemos uma solução para a equação. Tomamos por exemplo o terno (1,1,1)

x² + 1² + 1² = 3.x.1.1 ou seja, x=1 ou x= 2

Então, além de (1,1,1), o terno (2,1,1) também é solução da equação.

Considerando agora o terno (2,1,1) teremos então por exemplo que 2² + 1² + z² =


3.2.1.z, ou seja z=1 ou z=5.

Então além de (2,1,1), o terno (2,1,5) também é solução.


De modo geral escolhendo uma terna e escolhendo um dos três valores, sempre acharemos outro
valor que satisfaz também a equação. Ou seja de um terno (x 0 , y0 , z0) sempre obteremos um
outro terno (x1, y1, z1).

Escolhendo x0 teremos:

x² + y0² + z0 ² = 3.x. y0 .z0


x² - 3.x. y0 .z0 + y0² + z0 ² = 0

Com essa equação do 2º grau, sabemos então que a soma das raízes é dada por s = 3. y0 .z0

Então x1 > x0 se x0 < 3/2 . y0 .z0 e caso contrário x0 > 3/2 . y0 .z0 .
Com isso garantimos que a segunda raiz sempre será maior que a primeira que foi substituída na
equação, e consequentemente, existirá infinitas soluções para a equação.

29. (Gazeta Matemática, Romênia) Considere a equação a2x2 (b2 - 2ac)x + c2 = 0; onde a;b e c são
números inteiros positivos. Se n  é tal que p(n) = 0, mostre que n é um quadrado perfeito.

Solução:

Temos que:

p (n)  0  a 2 n 2  (b 2  2a c) n  c 2  0

Observe que essa é uma equação do segundo grau na incógnita n . Para resolver essa equação
encontremos o seu discriminante:

  [  (b 2  2ac) ]2  4a 2 c 2  b 4  4ab 2 c  4a 2 c 2  4a 2 c 2  b 4  4ab 2 c  b 2 (b 2  4ac)

Daí, podemos concluir que b 2 (b 2  4ac) é um quadrado perfeito, pois n  N é raiz de p (x) , ou seja,
p (n)  0 e a , b e c são inteiros positivos o que nos garante que b 2  4ac também é um quadrado
perfeito, já que b 2 também o é. Mas b 2  4ac é o discriminante da equação do segundo grau
b  b 
a x 2  b x  c  0 , onde suas raízes x'  e x"  , devido ao valor de  , são
2a 2a
números racionais.

Voltando para a resolução da equação inicial, a 2 n 2  (b 2  2a c) n  c 2  0 , temos que:

b 2  2a c  b 2 (b 2  4a c) b 2  2a c  b b 2  4 a c b  b  b  4a c c
2

n    
2a2 2a2 a 2a a
Mas,

b c  b  b 2  4a c  b  b 2  4a c
  x'  x" ,  x'  x" ,  x' e  x" , onde x' e x" são, como já
a a 2a 2a
vimos, as raízes da equação a x 2  b x  c  0 .

Portanto, fazendo as devidas substituições, temos que:

n'  ( x'  x" )  x'  x'  x"  ( x' ) 2  x'  x"  x'  x"  n'  ( x' ) 2

Ou:

n"  ( x'  x" )  x"  x'  x"  ( x" ) 2  x'  x"  x'  x"  n"  ( x" ) 2

Logo, como n é um número natural que é igual ao quadrado de números racionais, então esses
números racionais tem que ser inteiros e n é, portanto, um quadrado perfeito.

Questão 30: (Gazeta Matemática, Romênia) Sejam a, b Z. Sabendo que a equação


(ax – b)2 + (bx – a)2 = x, tem uma raiz inteira, encontre os valores de suas raízes.

Solução:

(ax – b)2 + (bx – a)2 = x

a2x2 – 2abx + b2 + b2x2 – 2abx + a2 = x

(a2 + b2)x2 – (4ab + 1)x + a2 + b2 = 0

Se o produto é igual a 1 e uma as raízes é inteira, então elas são inversas e:

4ab + 1 ≥ 2.(a2 + b2)

2a2 + 2b2 – 4ab ≥ 1

2.(a – b)2 ≥ 1

√ √

⇒ ⇒

31. (Gazeta Matemática, Romênia) Resolva a equação:

* +

Obs.: [x] é o menor inteiro maior ou igual a x.

Solução:
Por definição de [a], teremos que: x  , 2x2   e x2 + 1  .
A expressão acima equivale a dizer que x é o quociente da divisão euclidiana de 2x2 por x2 + 1, isto é,
equivale a dizer que:  r  {0, 1, ..., x, ..., x2}, tal que:
2x2 = (x2 + 1).x + r, daí podemos afirmar

– x3 + 2x2 – x = r
x.( – x2 + 2x – 1) = r

mas temos que x | r, então concluímos que r  {0, x, 2x, ..., x2}, o que nos permite dizer que r = .x
com   {0, 1, ..., x}. Segue que

x.( – x2 + 2x – 1) = .r
x.( – x2 + 2x – (1+)) = 0
Segue daí que x = 0 ou que ( – x2 + 2x – (1+)) = 0. Da segunda igualdade, termos que √ ,
mas como x   e   {0, 1, ..., x},  = 0, e daí, termos x = 1 como a segunda solução.

32. Demonstrar que:


(a) n4 + 4 não é primo se n > 1;
(b) generalize, mostrando que n4 + 4n não é primo, para todo n > 1.

a) não é primo se n > 1

Solução:

Considere a igualdade de Sophie Germain

a4 + 4b4 = (a2 + 2b2 + 2ab)( a2 + 2b2 - 2ab)

Tomando a = n e b = 1, temos:

n4 + 4 = (n2 + 2 + 2n)( n2 + 2 - 2n), assim, n4 + 4 é composto.

b) Generalize, mostrando que n4 + 4n não é primo, para todo n > 1.

Solução:

Se n for par, segue que n4 e 4n são pares, logo n4 + 4n não é primo.

Se n for ímpar, n é da forma 2m + 1, então, substituindo temos:

(2m + 1)4 + 42m + 1

(2m + 1)4 + 4  42m

(2m + 1)4 + 4  24m

(2m + 1)4 + 4 

Tomando a = 2m + 1 e b = 2m , o resultado é conseqüência da igualdade de Sophie Germain.


Questão 33: Para fazer 12 bolinhos, preciso exatamente de 100 g de açúcar, 50 g de manteiga, meio
litro de leite e 400 g de farinha. Qual a maior quantidade desses bolinhos que serei capaz de fazer com
500 g de açúcar, 300 g de manteiga, 4 litros de leite e 5 kg de farinha?

Solução:

Para fazer 12 bolinhos:


100g de Açúcar
50g de Manteiga
0,5 l de Leite
400g de Farinha.

Com:
500g de Açucar conseguimos fazer 60 bolinhos
50g de Manteiga conseguimos fazer 72 bolinhos
0,5l de Leite conseguimos fazer 96 bolinhos
400g de Farinha conseguimos fazer 150 bolinhos
Portanto o açúcar é o ingrediente limite e podemos fazer 60 bolinhos no
máximo.

34. Dadas as frações

qual é maior?

Solução:

Denominando x = , x +1 = e x + 2 = , teremos uma


expressão a qual iremos supor uma afirmação, de que a primeira fração é menor do que a segunda.

O que é verdade, logo, a primeira fração é menor do que a segunda!

35. Achar o maior valor inteiro positivo de n tal que n200 < 5300:

Solução:

Aplicando as propriedades das potências, temos que:

( ) ( )

Como a base é positiva , segue que:

n2 < 53

n<√

n < 11,2
Como n  , temos que o maior valor inteiro positivo será quando n = 11.

36. Achar o menor valor inteiro positivo de n tal que

Solução:

1 + 2 + 3 + ... + n > 55

( )

n2 + n – 110 > 0

n > 0 (positivo), logo, n = 11.

37. Nove cópias de certas notas custam menos de R$ 10,00 e dez cópias das mesmas notas (com o
mesmo preço) custam mais de R$ 11,00. Quanto custa uma cópia das notas?

Solução:

Denominando de x = valor de uma cópia, teremos que:

9x < 10 → x < 10/9


10x > 11 → x > 11/10
Analisando as desigualdades, x < 1,111... e x > 1,10, iremos concluir que x = 1,11.

Uma cópia custa R$1,11.

38. Se enumeram de 1 até n as páginas de um livro. Ao somar estes números, por engano um deles é
somado duas vezes, obtendo-se o resultado incorreto: 1.986. Qual é o número da página que foi
somado duas vezes?

Solução:

Considere k o número que foi somado duas vezes. O valor de k é um número natural maior ou igual a
1 e menor ou igual a n.
Assim:

i) 1986 – (1 + 2 + ... + n) ≥ 1 ii) 1986 – (1 + 2 + ... + n) ≤ n


( ) ( )
1986 – [ -≥1 1986 – [ -≤n

– n2 – n ≥ 3970 – n2 – n ≤ 3972

n2 + n – 3970 ≤ 0 n2 + 3n – 3972 ≥ 0

Resolvendo a inequação, encontramos: Resolvendo a inequação, encontramos:

- 63,5 ≤ n ≤ 62,5 n ≤ - 64,5 e n ≥ 61,5

As duas inequações apresentam o seguinte conjunto solução para n natural

61,5 ≤ n ≤ 62,5,

Segue que o único valor de compreendido nesse intervalo é 62.

Assim, o número de páginas do livro é 62 páginas

A soma das páginas seria:


( )
1 + 2 + 3 + ... + 62 = .

Como havia somado 1986, logo: 1986 – 1953 = 33.

A página somada duas vezes, foi a página 33.

39. Determine os valores de a para os quais a função quadrática ax2 – ax + 12 é sempre positiva.

Solução:

Para que a função seja sempre positiva deve obedecer:


a>0e<0
 = a2 – 4.a.12 < 0  0 < a < 48.

Para uma equação ter apenas resultados positivos é necessário que delta seja
menor que zero, e o coeficiente do termo quadrático positivo.

40. Ache os valores de x para os quais cada uma das seguintes expressões é positiva:

( ) ( ) ( )

a) Solução:

Pelo numerador temos que teremos uma imagem positiva para x > 0 e imagem negativa para x < 0.
Enquanto que para o denominador teremos uma imagem positiva para qualquer valor de x
(discriminante menor que zero)
Fazendo o estudo dos sinais, veremos que o valor que satisfará a condição do problema será:
S = { x  ℝ / x > 0}
b) Solução:

Pelo numerador temos que teremos uma imagem positiva para x > 3 e imagem negativa para x < 3.
Enquanto que para o denominador teremos uma imagem positiva para x > – 1 e teremos uma imagem
negativa para x < – 1.
Fazendo o estudo dos sinais, veremos que o valor que satisfará a condição do problema será:

S = { x  ℝ / x < – 1 ou x > 3}

c) Solução:

Pelo numerador temos que teremos uma imagem positiva para x < – 1 ou x > 1 e imagem negativa
para – 1 < x < 1.
Enquanto que para o denominador teremos uma imagem positiva para x < 0 ou x > 3 e teremos uma
imagem negativa para 0 < x < 3.
Fazendo o estudo dos sinais, veremos que o valor que satisfará a condição do problema será:

S = { x  ℝ / x < – 1 ou 0 < x < 1 ou x > 3}

41. Resolve r a equação:

[x]{x}+ x = 2{x}+ 10,

onde [x] denota a parte inteira de x. Por exemplo, [2;46] = 2 e [5;83] = 5. O número {x} é chamado
parte fracionária de x e é definido por {x} = x - [x].

Solução:

Considere x = [x] + {x} e substituindo na equação, temos:

|x|{x} + [x] + {x} = 2{x} + 10

|x|{x} + [x] – {x} = 10

{x} ( [x] – 1) = 10 – [x]


, -
{x} = , -
, da expressão temos que considerar que:

i) 0 < {x} ≤ 1, visto que a parte fracionária tem que ser um valor entre 0 e 1.

ii) [x] ≤ 10, pelo fato de que, se fosse maior que 10, teríamos uma parte fracionária negativa.

iii) 10 – [x] ≤ [x] – 1, isto é, o numerador tem que ser menor ou igual ao denominador.

Resolvendo essa inequação temos:

– [x] – [x] ≤ – 1 – 10

– 2 [x] ≤ – 11

[x] ≥ 5,5 , mas como [x] tem que ser inteiro, então [x] ≥ 6

Assim, 6 ≤ [x] ≤ 10.


, -
Substituindo em {x} = , -
, obtemos:
i) Para [x] = 6 , {x} = iv) Para [x] = 9 , {x} =

ii) Para [x] = 7 , {x} = = v) Para [x] = 10 , {x} =

iii) Para [x] = 8 , {x} =

Assim, a solução é : 6, 8; 7,5; ; 9,125 e 10

42. Mostre que entre os retângulos com um mesmo perímetro, o de maior área é um quadrado.

Solução:

Considere um retângulo cujos lados medem x e y. Temos que a área, A, desse retângulo é dada por
e seu perímetro, p, é dado por
.

Logo, . / .

E mais, . Portanto, para A máximo temos x = y.

43. Entre todos os triângulos isósceles com perímetro p fixado, ache as dimensões dos lados daquele
que possui a maior área.

Solução:

Denominando cada um dos dois lados congruentes de a e o terceiro lado de b e o perímetro de p,


teremos que:

p=a+a+b

p = 2a + b

Usando a fórmula de Herón temos que a área de um triângulo com perímetro p é dada pela expressão

√ . /. /. /

No caso específico do triângulo isósceles, ela ficará

√ . /. /. /

Analisando uma parte da fórmula: . /. /. /,

Temos que MG ≤ MA, logo,

√. /. /. /

O que nos permite concluir que


(√. /. /. /) ( )

(√. /. /. /) . /

. /. /. /

O maior valor da desigualdade que o primeiro membro atingirá será quando ele for igual ao segundo
membro, logo, substituindo na fórmula de Herón, teremos:


Logo a maior área possível é , a qual é atingida quando


. / . /⇔

Ou seja, quando o triângulo for eqüilátero.

44. (OBM Júnior 1993)

É dada uma equação do segundo grau x2 + ax + b = 0, com raízes inteiras a1 e b1. Consideramos a
equação do segundo grau x2 + a1x + b1 = 0. Se a equação x2 + a1x + b1 = 0 tem raízes inteiras a2 e b2,
consideram s a equação x2 + a2x+ b2 = 0. Se a equação x2 +a2x+b2 = 0 tem raízes inteiras a3 e b3,
consideramos a equação x2 + a3x+ b3 = 0. E assim por diante . Se encontramos uma equação com 
< 0 ou com raízes que não sejam inteiros, encerramos o processo. Por exemplo, se começamos com a
equação x2 = 0 podemos continuar o processo indefinidamente. Pede-se:

(a) Determine uma outra equação que, como x2 = 0, nos permita continuar o processo indefinidamente;

(b) Determine todas as equações do segundo grau completas a partir das quais possamos continuar o
processo indefinidamente.

Seja P0(x) = x2 + a0x + b0 = 0 uma equação com raízes inteiras a1 e b1.

Vamos definir a equação Pi(x) = x2 + aix + bi = 0 como sendo a equação obtida no i-ésimo passo do
procedimento descrito pelo enunciado, ou seja, ai e bi são as raízes inteiras da equação Pi – 1(x) = x2 +
ai - 1x + bi - 1 = 0, para i = 1, 2, ... .

Responderemos agora os dois itens.


a) Solução:

Seja P0(x) = x2 + kx = 0, onde k  .

Como se vê facilmente, podemos obter a partir de P0(x) = 0 as equações:

Pi(x) = x2 + (– 1)ikx = 0, i = 1, 2, ... .

b) Solução:

Nosso próximo passo será determinar todas as equações que nos possibilitem continuar o processo
indefinidamente.

Vimos no item (a) que as equações x2 + kx = 0, k  , têm essa propriedade, por isso suponhamos b0
≠ 0 em P0(x) = x2 + a0x + b0 = 0. Evidentemente bi - 1 = ai .bi, ou seja, bi divide bi – 1, o que mostra que,
para todo i inteiro positivo, bi divide b0.

Além disso, b0 possui um número finito de divisores, pois b0 ≠ 0.

Daí resulta que, se b0 ≠ 0 e o processo continua indefinidamente, pelo menos um dos divisores bj de b0
deve aparecer infinitas vezes como termo independente das equações do processo.

Digamos que bj volte aparecer como termo independente de uma equação P k(x) = x2 + akx + bk
= 0 para algum k > j.

Mas como bi divide bi – 1 para todo i inteiro positivo, temos |bi – 1| ≥ |bi| e daí |bj| ≥ |bj+1|≥ ... ≥ |bk| = |bj|, o
que mostra que |bi| = |bj| para i = j + 1, j + 2, ..., k, onde bi  { – bj, bj} para todo i > j.

Há dois casos a considerar:

( )
,
( )

( )
,
( )

No primeiro caso, observemos que é raiz de ( ) , o que implica que a outra raiz é – 1 e
( ) .

Se o termo independente de ( ) é , então 1 é raiz de ( ), o que implica que ( )


⇒ , o que é absurdo pois divide b0.

Se o termo independente de ( )é , então – 1 é raiz de ( ), o que mostra que ( )


⇒ ⇒ ( ) ⇒ ( ) , o que é uma equação cujas raízes
não são inteiras.

Vemos portanto que neste caso não é possível continuar o processo.

Uma análise inteiramente análoga a do primeiro caso permite-nos concluir, no segundo caso,
( ) que é uma equação onde uma das raízes é 1 ou – 1.
Se a raiz for – 1, então ( ) , o que é absurdo.

Se a raiz for 1, então ( ) ⇒ .

Logo ( ) , cujas raízes são 1 e – 2.

Assim podemos fazer ( ) para i ≥ j + 2, e repetir este procedimento


indefinidamente.

É imediata a verificação de que ( ) também implica que ( )


para 0 ≤ i ≤ j.

Segue que ( ) é a única equação com termo independente não nulo a partir da
qual podemos repetir o processo um número arbitrário de vezes.
Capítulo 3 – Soluções dos Exercícios

1. Encontre o resto que deixa

(a) 2001. 2002 . 2003 . 2004 + 20052 quando é dividido por 7;

O produto 2001. 2002 . 2003 . 2004 é divisível por 7 pois 2002  0 mod 7.

Como 2005  3 mod 7 temos que 20052  32 mod 7  2 mod 7.

Logo o resto será igual a 2.

(b) 2100 quando é dividido por 3;

Sabemos que 22  1 mod 3 e 2100  2 2   50


portanto 2100  150 mod 3  1 mod 3.

Então o resto será igual a 1.


(c) 1237156  34 
28
quando é dividido por 111.

Sabemos que 12371  50 mod 111; 123712  58 mod 111; 123718  584 mod 111 46 mod 111.


Como 1237156  123718 7
 46 7 mod 111  16 mod 111.

Portanto devemos encontrar o resto da divisão de 16  34  50 28 por 111.


28

Temos que 50 2  58 mod 111 ; como 50 28  50 2   14


então 50 28  5814 mod 111 .

Sendo 58 2  34 mod 111 logo 5814  58 2   7


 34 7 mod 111.

Se 34 7  34 3 . 34 4 e 34 3  10 mod 111 e 34 4  7 mod 111então 34 7  7.10 mod 111 .

Portanto temos um resto igual a 70.

2. Provar que o número n5 + 4n é divisível por 5 para qualquer número natural n.

Adicionando e subtraindo 5n à expressão n5 + 4n teremos então n5 + 4n + 5n – 5n = n5 – n + 5n.

Pelo pequeno teorema de Fermat temos que 5 n 5  n e 5 5n .

Sabemos que se a b e a c então a b  c .

Portanto 5 n 5  4n .
3. Prove que se n é ímpar

(a) n3 – n é divisível por 24;

Seja P  n 3  n  n  1.n.n  1 um número inteiro com n ímpar.

i) Sabemos que o produto de dois números consecutivos é múltiplo de 2, então P é divisível por 2.

ii) Sendo n ímpar temos que n  1 e n  1 são pares então P é divisível por 22.

iii) O produto de três números consecutivos é divisível por 3, então P é divisível por 3.

iv) Todo número divisível por 2 e 3 é divisível por 6, então P é divisível por 6.

v) Se P é múltiplo de 2 2 e de 6 então será múltiplo de 24.

(b) n2 – 1 é divisível por 8;

Seja P  n 2  1  n  1
. n  1 um número inteiro com n ímpar.

Sendo n  2k  1, para k  Z temos que P  n 2  1  2k  1  1.2k  1  1  2k.2k  2  4k.(k  1) .

i) Se k é par temos que P = 4k = 8m.

ii) Se k é ímpar k + 1 é par então temos P = 8m.

Portanto P será sempre múltiplo de 8.

(c) n2 + (n + 2)2 + (n + 4)2 + 1 é divisível por 12.

Seja P  n 2  n  2  n  4  1 um número inteiro com n ímpar.


2 2


Desenvolvendo temos P  n 2  n 2  4n  4  n 2  8n  16  1 = 3n 2  12n  21  3 n 2  4n 7 . 

Sendo n  2k  1, para k  Z temos que P  3 2k  1  42k  1 7 
2


P  3 4k 2  4k  1  8k  4 7  
 P  3 4k 2  12k  12  
 P  3 4 k 2  3k  3  

P  12 k  3k  3
2
  P  12m , m  Z, logo P é múltiplo de 12.

4. O número 21093 – 2 é divisível por 10932?

Como 1093 é primo e 2, 1093  1, temos, pelo Pequeno Teorema de Fermat, que 1093 21093  2 .


Sabemos que 21093  2  2. 21092  1 . 
Da aritmética dos restos temos que a p  b p mod p  a p  b p mod p 2 , com a, b, p  N e p primo.

Fazendo p = 1093 temos que a1093  b1093 mod 1093  a1093  b1093 mod 10932 .

Temos que 37  2187  2.1093  1  2 p  1 logo 37  2.1093  1 mod 1093

37  2 p  1 mod p  3   2 p  1
7 2 2
mod p 2   
314  4 p 2  4 p  1 mod p 2

Como 4 p 2  0 mod p 2 então 314  4 p  1 mod p 2 (1)

214  16384  15.1093  11  15 p  11

214  15 p  11 mod p    15 p  11


 214
2 2
mod p 2  
 2 28  225 p 2  330 p  121 mod p 2

2 28   330 p  121 mod p 2 multiplicando por 32 temos 32.2 28   2970 p  1089 mod p 2 .

Como 2970p = 1093p + 1876p + p; 1089 = 1093 – 4;  p 2  0 mod p 2 e sendo p = 1093 temos:

32.2 28   1876 p  4 mod p 2 que dividido por 4 resulta em 32.2 26  469 p  1 mod p 2

3 .2 
2 26 7
 469 p  1 mod p 2
7
 314.2182  469 p  1 mod p 2
7

Desenvolvendo o binômio e excluindo as congruências 0 mod p 2 restará:

314.2182  3283 p  1 mod p 2  314.2182  3.1093 p  4 p  1 mod p 2

 
314.2182   3 p 2  4 p  1 mod p 2  314.2182  4 p  1 mod p 2 (2)

Usando as expressões (1) e (2) temos 2182  1 mod p 2  2    1


182 6 6
mod p 2 

21092  1 mod p 2 multiplicando por 2 temos que 21093  2 mod p 2 .

Logo 1093 21093  2 .

5. Prove que 9999941234567890 1 é divisível por 333331.

Considere P  9999941234567890 1 .
1234567889
Podemos escrever P  9999941234567890 1  999994  1.  999994 k .
k 1

1234567889
Logo P  999993. 
k 1
999994 k  999993.M , sendo M  N.

Como 999993  3 . 333331 então P  3 . 333331. M  333331. S , sendo S  N.

Portanto P é múltiplo de 333331.

6. O número N = 42005 + 20054 é primo?

N  2005 4  4 2005  2005 4  4.4 2004  2005 4  4. 4 501 .   4

Fazendo a  2005 e b  4 501 , segue pela desigualdade de Sophie Germain que N é composto.

7. Demonstre que o número 1000


 ...
001
 é composto.
2006 zeros

1000...001 = 1000 00 + 1 = 10 2007  1


...

2007 zeros


10 2007  1 = 10 669  133


Fazendo a  10 669 
3
e b  13 temos que a 3  b 3  a  b.a 2  a b  b 2 .

 
Como a 3  b 3  a  b. a 2  a b  b 2 é composto então 1000
 ...
001
 também é composto.
2006 zeros

8. Utilizando o fato de que o resto de um quadrado quando dividido por 4 só pode ser 0 ou 1, dê uma
outra solução para o problema do Exemplo 3.54.

Seja P = aabb um número natural quadrado perfeito a  {1, 2, ..., 9} e a  b.

Temos que P = 4q + 0 ou P = 4q + 1.

Para P = aabb  0 mod 4 temos que b = 0; b = 4 ou b = 8.

Para b = 8, P não é quadrado perfeito, pois nenhum quadrado perfeito termina em 8.


Para b = 0, P não é quadrado perfeito, pois teríamos P  aa .100 e não existe nenhum quadrado
perfeito da forma aa.

Para b = 4, P = aa44 temos que 30  P  100 . Por tentativa temos que 882 = 7744, portanto a = 7.

Para P = aabb  1 mod 4  4 11b  1 , logo b = 3 ou b = 7. Nestes casos P também não é quadrado
perfeito, pois nenhum quadrado perfeito termina em 3 ou em 7.

9. Dados três inteiros, x, y, z, tais que x2 + y2 = z2, mostre que x e y não são ambos ímpares e que xy é
múltiplo de 6.

I) Suponha por absurdo x e y ímpares. Sendo x = 2t + 1 e y = 2v + 1 e {t, v}  Z.

 
Como x2 + y2 = z2 então 4t 2  4t  1  4v 2  4v  1  z 2  4 t 2  t  v 2  v  2  z 2  4m + 2 = z2,
mostrando que z2 é par logo z também é par.

Seja z= 2u com u  Z, então z2 = 4u2.

Portanto 4m + 2 = 4u2 dividindo por 2 temos 2


 
m 1  2
u 2 o que é um absurdo.
ímpar par

II) Mostrar que xy é múltiplo de 3

Suponha x e y não múltiplos de 3. Logo z = 3k, k  Z.

Seja x  3q1  r1 e y  3q2  r2 com r1 , r2   1, 2.

Como x2 + y2 = z2 temos que 9q1  6q1r1  r1  9q2  6q2 r2  r2  9k 2 então 3 r1  r2 .


2 2 2 2 2 2

Como x e y não são ambos ímpares e xy é múltiplo de 3 então xy é múltiplo de 6.

10. Demonstre que o quadrado de um inteiro é da forma 8n ou 8n+1 ou 8n + 4.

Seja m um número inteiro. Podemos escrever m = 4k; m = 4k + 1; m = 4k + 2 ou m = 4k + 3.

Se m = 4k então m2 = 16k2 = 8(2k2) = 8n.

Se m = 4k + 1 então m2 = (4k + 1)2 = 16k2 + 8k + 1 = 8 (2k2 + k) + 1 = 8n + 1.

Se m = 4k + 2 então m2 = (4k + 2)2 = 16k2 + 16k + 4 = 8 (2k2 + 2k) + 4 = 8n + 4.

Se m = 4k + 3 então m2 = (4k + 3)2 = 16k2 + 24k + 9 = 16k2 + 24k + 8 + 1 = 8(2k2 + 3k + 1) + 1 = 8n + 1.

Outra solução:
 
Seja p = 4q + r a divisão de p por 4. Assim p 2  16q 2  8qr  r 2  8 2q 2  qr  r 2 .

Logo p 2  r 2 mod 8 , como r {0, 1, 2, 3} então r2  {0, 1, 4}.

Portanto p 2  8n ou p 2  8n  1 ou p 2  8n  4 .

11. Três números primos p; q e r, maiores que 3, formam uma progressão aritmética, ou seja, q = p + d
e r = p + 2d. Prove que d é divisível por 6.

Para provar que d é divisível por 6, devemos mostrar que 2|d e 3|d.

Como todos os primos maiores que 3 são ímpares e, supondo que d seja ímpar, temos que q é par,
pois, a soma de dois números ímpares resulta em um número par, o que é um absurdo, pois, q é
primo. Assim, d é par, isto é, 2|d.

Supondo que d não seja divisível por 3, então d é da forma 3k + 1 ou 3k + 2, com k N.

Para d = 3k + 1, verificamos que:

p=p  q = p + d = p + 3k + 1  r = p + 2(3k + 1) = p + 3(2k) + 2

Para d = 3k + 2, verificamos que:

p=p  q = p + d = p + 3k + 2  r = p + 2(3k + 2) = p + 3(2k + 1) + 1

Assim, pelo Lema dos restos, na divisão por 3 os valores de q e r são da forma p + 1 e p + 2.

Logo, sendo p, q e r três números consecutivos, ou seja, p, p + 1 e p + 2, então, pelo menos um deles
é múltiplo de 3, mostrando que 3| d.

Portanto, como 2|d e 3|d, então 6|d.

12. Demonstrar que existem infinitos números primos da forma 4m + 3 e da forma 6m + 5, onde m  Z.

Um número inteiro P qualquer pode ser inscrito como P  4q  r , com r  {0, 1, 2, 3}. Se P é primo
então r = 1 ou r = 3.

Suponha que existam finitos primos na forma 4m + 3 e seja A o conjunto de todos os primos dessa
forma, ou seja A  4m  1, m  N   p0  2, p1  5, p2 ,..., pn .

Observe que o conjunto A é fechado multiplicativamente.

Suponhamos, por absurdo, que existam apenas um número finito de primos p1 < ... < pk da forma
4m 3 , com m > 1. O número P = 4(p1.p2....pk) + 3 não é divisível por nenhum dos números primos 3,
p1, ..., pk e, portanto, sua decomposição em fatores primos só pode conter primos da forma 4m + 1, o
que é uma contradição, pois P é da forma 4m + 3.
Considere um número natural S qualquer. Podemos escrevê-lo na forma 6m  r , r  0, 1,...,5.

Se o número S é primo então r  1, 5 .

Suponhamos, por absurdo, que existam apenas um número finito de primos p1 < ... < pk da forma
6m 5 , com m > 1. O número S = 6(p1.p2....pk) + 5 não é divisível por nenhum dos números primos 5,
p1, ..., pk e, portanto, sua decomposição em fatores primos só pode conter primos da forma 6m + 1.

Como o conjunto B = {6n + 1; nN} é fechado multiplicativamente, então S é da forma 6m + 1, o que é


uma contradição. Logo existem infinitos números primos da forma 6m + 5.

13. Encontrar o último dígito dos números (resto da divisão por 10)

(a) 19892005

Sabemos que 1989 2005  1989 . 1989 2  


1002
.

1989  9 mod 10 e 1989 2  9 2 mod 10  81 mod 10  1 mod 10

1989 
2 1002
 11002 mod 10  1 mod 10 .


1989 205  1989 . 1989 2 
1002
 9 . 1 mod 10  9 mod 10

Logo o dígito da unidade é 9.

(b) 777777 + 250

34  81  1 mod 10 e 27  7 mod 10

777  7 mod 10  777 777  7 3   259


 
 343259  33.3256  27. 34
64
 7.1 mod 10  7 mod 10

2 5  2 mod 10  
 2 50  2 5
10
 210  1024  4 mod 10

Logo 777 777  250  7  4 mod 10  1 mod 10 , então o algarismo da unidade é 1.

(c) 1 + 22 + 32 + ... + 20052.

Sabemos que 1  11  21  ...  1991  1 mod 10 ; 2  12  22  ...  1992  2 mod 10 , ou seja números
de mesmo final possuem mesmos restos mod 10 e sabemos, também, que todo número quadrado só
termina em 0, 1, 4, 5, 6 ou 9.

Dividindo 2000 em grupos de 10 temos 200 grupos.

Portanto temos
i) de 1 até 2000 temos 200.1  4  9  6  5 6  9 4 1 0  200.45  9000  0 mod 10

ii) de 2001 até 2005 temos 1  4 9 4  5  25  5 mod 10

Portanto o algarismo da unidade é 5.

14. Prove que a soma dos quadrados de cinco números consecutivos não é um quadrado perfeito.

Sejam n  2; n  1; n; n  1; n  2 números naturais consecutivos.

Somando os quadrados n  2  n  1  n 2  n  1  n  2 temos 5n 2  10 .


2 2 2 2

Se n é par (n = 2k) temos 4k '  2 . Se n é ímpar (n = 2k + 1) temos 4k ''  3 .

Como um quadrado perfeito é da forma 4k ou 4k  1 .

Portanto a soma dos quadrados de cinco números consecutivos não é um quadrado perfeito.

15. Prove que 1000


...
005000
 ...
001 não é um cubo perfeito.

100 zeros 100 zeros

Se n é inteiro podemos escrever n  3q  n 3  9k , k   ou n  3q 1  n 3  9k 1, k   ou


n  3q  2  n 3  9k  8, k   .

Seja X  1000
...
005000
  001 então podemos escrever X  10 202  5.10101 1 .
...

100 zeros 100 zeros

Pelo lema dos restos temos que 10 202  1 mod 9 ; 5. 10101  5.1 mod 9 e 1  1 mod 9 .

Logo X deixa resto 1 + 5 + 1 = 7 quando dividido por 9, portanto X não é cubo perfeito.

16. Seja b um inteiro positivo. Enuncie e prove o critério de divisibilidade por b no sistema de
numeração de base b.

Proposição: Se P  an an1 ...a1a0 b  Z com ai  0, 1, 2,..., b  1, i  0, 1, 2,..., n, então b P 
a0  0 .

() Seja P um número escrito na base b, logo P  an b n  an1b n1  ...  a1b  a0 .

Podemos escrever P  b.(an b n1  an1b n2  ...  a1 )  a0 .

Pela proposição 3.3 temos que b P , logo b b.(an b n1  an1b n2  ...  a1 )  a0 .
É fato que b b.(an b n1  an1b n2  ...  a1 ) então b também deve dividir a 0 . Como 0  a0  b  1
temos necessariamente que a0  0 .

() Seja P  an b n  an1b n1  ...  a1b um número em base b, logo


P  b.(an b n1  an1b n2  ...  a1 )  b . q  b P

17. Prove que os números

1 1 1
(a)  n  1   ...  , com n > 1,
2 3 n

Considere p o maior número primo tal que p < n, então,

2  3  ...  p  ...  n  1  3  4  ...  p  ...  n  ...  1  2  ...   p  1   p  1  ...  n  ...  1  2  ...  p  ...  n  1
n 
1  2  3  ...  p  ...  n

Fazendo

m  2  3  ...  p  ...  n  1 3  4  ...  p  ...  n  ...  1 2  ...   p  1  p  1 ...  n  ...  1 2  ...  p  ...  n 1 e
logo . m
k  1 2  ...   p  1  p  1 ...  n n 
pk

Como p não divide 1  2  3  ...   p  1 p  1...n , que é uma parcela de m, então pk também não divide
1  2  3  ...   p  1 p  1...n portanto, n não é inteiro.

1 1 1
(b)  n   ...  , com n > 0, não são inteiros.
3 5 2n 1

Considere p o maior número primo tal que p < n.

Então,

5  7  ...  p  ...  2n  1  3  7  ...  p  ...  2n  1  ...  3  5  ...   p  2   p  2  ...  2n  1  ...  3  ...  p  ...  2n  1
n 
3  5  7  ...  p  ...  2n 1

Fazendo
m  5  7  ...  p  ...  2n  1  3  7  ...  p  ...  2n  1  ...  3  5  ...   p  2   p  2  ...  2n  1  ...  3  ...  p  ...  2n  1

e k  3  5  7  ...   p  2 p  2  ...  2n  1 então  n 


m
.
pk

Como p não divide 3  5  7  ...   p  2 p  2  ...  2n  1 , que é uma parcela de m, então pk também
não divide 3  5  7  ...   p  2 p  2  ...  2n  1 , portanto, n não é inteiro.

18. Considere o polinômio p(n)  am n m  am1 n m1  ...  a0 de grau m > 1 com coeficientes inteiros e
nN. Prove que p(n) é um número composto para infinitos valores de n.

Sugestão: Use o fato de que existe a N tal que   p(a)  1 e mostre que  divide p  k  a  , para
todo k  Z.

Sabemos que   p(a) e temos que p(a)  am a m  am1 a m1  ...  a0 .

Calculando p  k  a  em p(n)  am n m  am1 n m1  ...  a0 temos:

p( k a )  am ( k a ) m  am1 ( k a ) m1  ...  a0 que, após desenvolver os binômios de cada


parcela, restará p( k a )  am a m  am1 a m1 ...  a0  k  k   m m1
 ...  k .
Como a N, a primeira parcela representa p(a)   e como k  Z, a segunda parcela representa
uma soma de potências de  que pode ser escrita como Q . , pois é múltiplo de .

Então p( k a )    Q .   1  Q  T .

19. Dizemos que um conjunto An formado por n inteiros positivos escritos no sistema binário (base 2) é
regular se, para qualquer s inteiro não negativo a quantidade de números de A n que contemplam 2s na
representação binária é par. Dizemos que An é irregular se, pelo menos para algum s, este número é
ímpar. Demonstre que um sistema irregular pode se converter em regular excluindo-se apenas um
único elemento do mesmo, e, um sistema regular pode se converter em irregular excluindo-se qualquer
um dos seus elementos.

20. Seja n um inteiro positivo. Demonstrar que todos os coeficientes do desenvolvimento do binômio
de Newton (a + b)n são ímpares se, e somente se, n é da forma 2 s  1 .
21. Prove que se (x0, y0) é uma solução da equação diofantina linear ax – by = 1, então a área do
1
triângulo cujos vértices são (0, 0), (b, a) e (x0, y0) é .
2
Se (x0, y0) é solução da equação ax – by = 1, então ax0 – by0 = 1.

1
Sabemos que a área de uma região triangular, no plano, pode ser dada por A  . D , sendo D o
2
determinante formado com as coordenadas dos vértices.

0 0 1
1 1 1 1
Então A  x0 y0 1 = ax0  by 0 = 1 =
2 2 2 2
b a 1

22. Qual é a menor distância possível entre dois pontos (x1, y1) e (x2, y2), com coordenadas inteiras,
situados sobre a reta definida pela equação diofantina ax + by = c?

Temos que ax1  by1  c e ax2  by 2  c

Então ax1  by1  ax2  by 2 logo b  y1  y 2    a x1  x2   y1  y 2   x1  x2  .


a
b

Como a distância entre dois pontos no plano é dada por d  x1  x2 2   y1  y2 2 temos, então que
2
 a 
2
d x1  x2  2
  x1  x 2   d x1  x2 2  a 2 x1  x2 2 
 b  b
 2
  a 2 b 2 
d x1  x2 2 1  a 2   d  x1  x 2  2
 .
 b   b 

Se (x1, y1)  (x2, y2) e as coordenadas são inteiras temos d  0 e x1  x2  1.

 a 2 b 2 
A distância mínima ocorre quando x1  x2  1 então d min   2
 .
 b 
Capítulo 4- Soluções dos Exercícios

1. Seja C um conjunto formado por cinco pontos de coordenadas inteiras no plano. Prove que o ponto
médio de algum dos segmentos com extremos em C tem também coordenadas inteiras.

Solução:

Seja C = { P1, P2, P3, P4, P5 } o conjuntos dos 5 pontos de coordenadas inteiras no plano.
Temos as seguintes possibilidades:

P1= { par, par}; P2= { par ímpar}; P3= {ímpar, par} e P4= {ímpar, ímpar}.

O ponto médio entre dois pontos A e B é dado por M( xM, yM) , onde:

Observe que para o ponto médio tenha coordenadas inteiras xA e xB, é necessário que sejam ambos
pares ou ambos ímpares, o mesmo ocorre com yA e yB.
Dessa forma, os pontos P1, P2, P3, P4, P5 representam os pombos, e a paridade de suas coordenadas
representam as casas, onde não pode haver outra possibilidade.
Pelo P.C.P., P5 terá a paridade de suas coordenadas igual a um dos pontos P1, P2, P3, P4 .
Portanto, o ponto médio de algum dos segmentos com extremidades em C tem também coordenadas
inteiras.

2. O conjunto dos dígitos 1, 2, ..., 9 é dividido em três grupos. Prove que o produto de alguns dos
grupos deve ser maior que 71.

Solução:

Sejam 1, 2, 3, os produtos dos elementos do grupo 1, do grupo 2 e do grupo 3, respectivamente.


Suponha que:

1 ≤71
2 ≤71
3 ≤71

Daí, como 1 > 0, 2 > 0, 3 > 0, temos:

362880 = 1 ∙ 2 ∙ 3 ≤357911

ou seja,

362880 ≤357911 (absurdo!!!)

Portanto, o produto de alguns dos grupos deve ser maior que 71.

3. Prove que se N é ímpar então para qualquer bijeção

p : I N  IN

do conjunto IN = {1, 2, ... ,N} o produto P(p) = (1 – p(1))(2 – p(2)) ... (N – p(N)) é necessariamente par.
(Dica: O produto de vários fatores é par se, e somente se, um dos fatores é par.)
Solução:

Se N é ímpar então é da forma N = 2k + 1. Se P(p) fosse ímpar, todos os termos (i – p(i)) seriam
ímpares para todo 1 i N. Sendo assim (N – p(N)) = (2k + 1 – p(2k + 1)) =
2k’ + 1 p(2k + 1) = 2(k – k’), o que implica que P(N) é par, logo P(p) não é ímpar.

4. Dado um conjunto de 25 pontos no plano tais que entre quaisquer 3 deles existe um par com
distância menor que 1. Prove que existe um círculo de raio 1 que contém pelo menos 13 dos 25 pontos
dados.

Solução:

(i)

Inicialmente escolhemos dois pontos P e Q do plano cuja distância de P a Q é maior


que 2, traçamos dois círculos C1 e C2 de raio 1 e centros P e Q respectivamente.
Se todos os pontos estiverem no interior de um dos dois círculos o problema está resolvido. Caso
contrário teríamos um dos pontos exterior aos dois círculos o que contraria o fato de que entre
quaisquer 3 deles um par existe um par com distância menor que 1.

(ii)

Vamos analisar a situação os círculos C1 e C2 possuem intersecção é não vazia, ou seja, C1 e C2


possuem intersecção possui d pontos, com isso, cada círculo tem pelo menos 23 – d pontos. Assim,
cada região que não é intersecção entre os dois círculos tem na “pior” das hipóteses
com 0 ≤ d ≤ 23. Se d = 0, temos a situação (i), caso contrário, um dos círculos terá pelo
menos 13 pontos.
.

5. Prove que entre quaisquer 5 pontos escolhidos dentro de um triângulo eqüilátero de lado 1 sempre
existe um par destes cuja distância não é maior que 0,5.

Solução:

Dividindo o triângulo equilátero de lado 1 em quatro triângulos eqüiláteros de lado 1/2 como a figura
que se segue.
Logo, pelo PCP pelo menos dois pontos devem estar no mesmo triângulo, uma vez que temos 4
triângulos e 5 pontos.

Assim, como a maior distância entre dois pontos que estão num triângulo equilátero ocorre quando
estes estiverem no vértice do triângulo e como os 4 triângulos menores tem lados ½, então pelo menos
dois de seus pontos estão a uma distância de no máximo ½.
.

6. Marquemos todos os centros dos 64 quadrinhos de um tabuleiro de xadrez de 8x8. É possível cortar
o tabuleiro com 13 linhas retas que não passem pelos pontos marcados e de forma tal que cada
pedaço de recorte do tabuleiro tenha no máximo um ponto marcado?

Solução:

Temos marcados no tabuleiro 64 pontos, estando alinhados em 8 linhas e 8 colunas. Para separarmos
os 8 pontos pertencentes as 8 colunas são necessárias 7 linhas e para dividirmos os 8 pontos
pertencentes as 8 linhas são necessárias 7 linhas, totalizando 14 linhas. Caso sejam usadas 13 linhas
poderemos dividir o tabuleiro em 56 espaços, como temos 64 pontos, pelo Princípio da Casa dos
Pombos, teremos pelo menos 8 casas com 2 pontos. Logo, não é possível cortar o tabuleiro com 13
linhas retas que não passem pelos pontos marcados e de forma tal que cada pedaço de recorte do
tabuleiro tenha no máximo um ponto marcado.

7. Prove que existem duas potências de 3 cuja diferença é divisível por 1997.

Solução:

Existem 1997 possíveis restos pela divisão por 1997. Considere a sequência das potências de 3: 30,
3¹, 3², 3³, ...,31997. Esta sequência é composta de 1998 números.
Portanto, pelo PCP, dois desses, digamos 3n e 3m, com n > m, têm mesmo resto quando divididos por
1997. Logo, a sua diferença 3n– 3m é divisível por 1997.

8. São escolhidos 6 números quaisquer pertencentes ao conjunto

A = {1, 2, ...., 10}.

Prove que existem dois destes seis números cuja a soma é ímpar.

Solução:

Sabemos que a soma de dois números naturais é ímpar se, os números não tiverem a mesma
paridade, isto é, se um for par e o outro for ímpar.
Como o conjunto A tem 5 números pares e 5 números ímpares e devemos escolher 6 números
quaisquer, pelo PCP, teremos dentre os seis números escolhidos, pelo menos um número par e um
número ímpar escolhido, onde a soma é ímpar.

9. Seja x um número real arbitrário. Prove que entre os números

x, 2x, 3x, ..., 101x

existe um tal que sua diferença com certo número inteiro é menor 0,011.

Solução:

Seja x um número real tal que, para certo inteiro  se tenha a.x  [, +1], para a  {1, 2, ...,
101} e x  .
Considere os conjuntos I = {x0 = 1, x1, x2, ..., x99, x100 =  +1} com x1< x2< ...< x99 <
x100, e xi – xi – 1 = 0,01,  i  {1, 2, ..., 100}.

Considere os intervalos Ii = [xi – 1, xi] e x =  ou 101.x =  + 1, nada há a demonstrar.

Suponha que x ≠  e 101.x ≠  + 1.

Temos # { Ii } = 100 e # { x, 2x, 3x, ..., 101x} = 101. Logo, temos 101 elementos para acomodar em 100
intervalos de comprimento 0,01. Segue que teremos dois números em um único intervalo. Se x > 0,
serão 100x e 101x. Se x < 0, serão x e 2x. Em qualquer caso teremos dois números num mesmo
intervalo, no qual um dos extremos é inteiro. Logo devemos ter x –  < 0,01 < 0,011 ou  + 1 – 101x
< 0,01 < 0,011.

10. Mostre que entre nove números que não possuem divisores primos maiores que cinco, existem
dois cujo produto é um quadrado.

Solução:

Seja A= {x1, x2, x3, x4, x5, x6, x7, x8, x9} o conjunto de nove números que não possuem divisores primos
maiores que cinco. Os números que não possuem divisores primos maiores que cinco são da forma xI
= 2.3.5.
Para que o produto de dois números xi.xj seja um quadrado, devemos ter xI = 2.3.5 e xj = 2’.3’.5’,
com i,j  {1, 2, 3, 4, 5, 6, 7, 8, 9}, ou seja, xi.xj = (2.3.5 ).( 2’.3’.5’ ) = 2+’.3+’.5+’, onde  + ’, 
+ ’ e  + ’ necessariamente devem ser pares, temos as seguintes possibilidades:
Possibilidades  e ’  e ’  e ’  + ’,  + ’,  + ’
1ª Pares Pares Pares Pares
2ª Pares Pares Ímpares Pares
3ª Pares Ímpares Pares Pares
4ª Pares Ímpares Ímpares Pares
5ª Ímpares Pares Pares Pares
6ª Ímpares Ímpares Ímpares Pares
7ª Ímpares Pares Ímpares Pares
8ª Ímpares Ímpares Ímpares Pares

Então existem 8 possibilidades para que o produto seja quadrado. Pelo P.C.P., as 8 possibilidades são
as casas e os 9 elementos de A são os pombos, e consequentemente, dentre os 9 elementos de A, o
produto de pelo menos dois deles é um quadrado.

11. Um disco fechado de raio um contém sete pontos, cujas distâncias entre quaisquer dois deles é
maior ou igual a um. Prove queo centro do disco é um destes pontos.

Solução:

12. Na região delimitada por um retângulo de largura quatro e altura três são marcados seis pontos.
Prove que existe ao menos um par destes pontos cuja distância entre eles não é maior que √ .

Solução:

Dividimos o retângulo em quadrados de lado unitário, como mostra a figura abaixo. Posicionamos 4
destes pontos nos vértices do retângulo maior. Em seguida posicionamos o quinto ponto no centro do
retângulo maior (onde do centro aos outros teremos 2,5 > √ ). Ao acrescentarmos o sexto ponto, em
qualquer região, ela não será maior que 2 em relação a 2 pontos mais próximos (lembrando que 2 <
√ ).

13. Seja a um número irracional. Prove que existem infinitos números racionais r = p/q tais que |a – r|
j< 1/q2.

Solução:

14. Suponha que cada ponto do reticulado plano é pintado de vermelho ou azul. Mostre que existe
algum retângulo com vértices no reticulado e todos da mesma cor.

Solução:

Se imaginarmos 3 linhas horizontais, as quais possuam 3 ponto de cores alternadas (vermelho ou


azul) e alinhados sobre uma reta perpendicular às horizontais; para cada paralela ao alinhamento
desses pontos teremos 2x2x2 = 8 possibilidades de disposição de cores aos pontos de intersecção
das paralelas com as horizontais. Então, sabemos que após 8 retas paralelas irá repetir algum padrão
das 8 possibilidades, e iremos ter o possível retângulo.
15. Um certo livreiro vende pelo menos um livro por dia. Sabendo que o livreiro vendeu 463 livros
durante 305 dias consecutivos, mostre que em algum período de dias consecutivos o livreiro vendeu
exatamente 144 livros.

Solução:

A soma de livros vendidos do 1º ao último dia é 463. Considere Sn como a soma dos livros vendidos do
1º dia ao dia n, com 1 305. Temos que:

1 S1 S2 <S3 <S4 < S305 = 463.

Note S4- S1 é a quantidade de livros vendidos entre o 1º e o 4º dia. Considere agora 1 p


Devemos mostrar que existe p e q tal que Sp- Sq= 144.

Observe que a seqüencia abaixo é crescente, pois é vendido por dia pelo menos 1 livro.

S1 S2 < S3 < S4 < S305 (I), logo

144+S1 144+S2 <144+S3 <144+S4 < S305 (II)

Das sequencias (I) e (II) temos:

 305+305= 610 números e


 463+144= 607 valores possíveis.

Pelo PCB, os números equivalem aos pombos e os possíveis valores as casas, logo S p seqüencia (I)
e Sq+ 144 à seqüencia (II).

Portanto em algum período de dias consecutivos o livreiro vendeu exatamente 144 livros.
Capítulo 5 – Soluções dos Exercícios

1. De quantas maneiras podemos escolher três números distintos do conjunto I50 = {1, 2, 3, ..., 49, 50 }
de modo que sua soma seja

a) um múltiplo de 3?

Devemos escolher números cujas classes de restos são do tipo 0 – 0 – 0 ou 1 – 1 – 1 ou 0 – 1 – 2 ou


2 – 2 – 2.

Classe de resto 0 são 16 números, de resto 1 ou 2 são 17 números cada.

16 
Para escolher três números de resto 0 num total de 16 temos    560 maneiras.
3
 

17 
Para escolher três números de resto 1 ou 2 num total de 17 temos 2 .    2 . 680  1360 maneiras.
3
 

Para escolher um número de resto 0, um número de resto 1 e um número de resto 2 temos, pelo
principio multiplicativo, 16 . 17 . 17 = 4624 maneiras.

Logo teremos 560 + 1360 + 4624 = 6544 maneiras distintas de escolher os três números cuja soma
será múltiplo de três.

b) um número par?

 25 
Podemos escolher três números pares num total de 25, ou seja,    2300 maneiras.
3
 

 25 
Podemos escolher dois números ímpares num total de 25, ou seja,    300 maneiras e um número
2
 
par num total de 25. Portanto teremos 300 . 25 = 7500 maneiras.

Finalmente pelo principio aditivo teremos um total de 2300 + 7500 = 9800 maneiras distintas de
escolher os três números cuja soma será par.
2. Considere o conjunto In = {1, 2, 3, ..., n – 1, n}. Diga de quantos modos é possível formar
subconjuntos de k elementos nos quais não haja números consecutivos?

Considerando um caso particular se o conjunto fosse {1, 2, 3, 4, 5, 6}, teríamos 4 opções para
formarmos 3 subconjuntos onde não existiriam números consecutivos. Seriam os seguintes
subconjuntos: {1, 3, 5} {1, 3, 6} {1, 4, 6} {2, 4, 6}

É lógico que este processo de enumeração é exaustivo e nada prático, então, vamos tentar
generalizar.

Usaremos o símbolo + para os elementos que farão parte do subconjunto e o símbolo - para os que
não farão parte dele.

Para o exemplo dado, com um conjunto de 6 elementos e subconjuntos de 3 elementos, teríamos 3


símbolos + e 3 símbolos - que, em cada subconjunto não poderiam estar seguidos.

Para o subconjunto {1, 3, 5}, a simbologia seria: + - + - + -.

Devemos perceber que, para 6 elementos, ficam definidos 7 posições possíveis (n + 1), conforme
figura abaixo.

- - -

Fixando os 3 lugares que seriam preenchidos pelos elementos que não farão parte do subconjunto,
sobrariam 4 posições (n – k +1) para serem escolhidas 3 para serem preenchidas pelos que farão
parte do subconjunto.

Note que, se temos 3 elementos que não vão participar do subconjunto, temos 3 + 1 (n – k +1)
posições para serem ocupadas pelos outros 3 elementos, que farão parte do subconjunto. Logo, em
nosso exemplo, temos uma única posição para os não participantes (-) e C4,3 para os participantes (+)
do subconjunto.

Então, generalizando, teremos para n elementos C n k 1, k .

3. Considere as letras da palavra PERMUTA. Quantos anagramas de 4 letras podem ser formados,
onde:

a) não há restrições quanto ao número de consoantes ou vogais?

b) o anagrama começa e termina por vogal?

c) a letra R aparece?

d) a letra T aparece e o anagrama termina por vogal?

a) 7 . 6 . 5 . 4 = 840 anagramas b) 3 . 5 . 4 . 2 = 120 anagramas

c) São 4 posições para colocar a letra R. Depois temos 6 . 5 . 4 = 120 maneiras de escolher as outras
três letras. Portanto temos 4 . 120 = 480 anagramas com a letra R.
d) São três posições para colocar a letra T. Temos 3 . (5 . 4 . 3) = 180 anagramas com a letra T que
terminam em vogal.

4. Calcular a soma de todos os números de 5 algarismos distintos formados com os algarismos 1; 3; 5;


7 e 9.

Teremos um total de 120 números todos da forma a.10 4 b.10 3 c.10 2  d .10  e . Sendo que a, b, c,
d, e podem assumir os valores 1, 3, 5, 7 e 9.

Somando esses algarismos teremos 1 + 3 + 5 + 7 + 9 = 25 (soma dos valores absolutos que


corresponde à soma quando cada um deles estiver em uma ordem fixa). Como são 24 números com
os algarismos ocupando uma posição fixa teremos 25 x 24 = 600.

Para somar todos os números devemos usar a decomposição do total que será dada por

(a  b  c d e).10 4  (a  b  c d e).103  (a  b  c d e).10 2  (a  b  c d e).10  (a  b  c d e)


multiplicado por 24. Como (a + b + c+ d + e) x 24 = 600, temos por soma o produto
 
600. 10 4  103  10 2  101  1 = 600 x 11111 = 6666600

5. Quantos números podem ser formados pela multiplicação de alguns ou de todos os números 2; 2; 3;
3; 3; 5; 5; 6; 8; 9; 9?

Tais números são do tipo 2 a.3b.5c , sendo 0  a  6; 0  b  8 e 0  c  2. Como são 7, 9 e 3


possibilidades para a, b e c, respectivamente, então são 7 . 9 . 3 = 189 possibilidades para 2 a.3b.5c .
Mas 1, 2, 3 , 5, 8, 38, 38 . 5, 38 . 52 , 26 , 26 . 5 e 26 . 52 não podem ser obtidos como pede o texto.
Assim, são 178 desses números.

6. Entre todos os números de sete dígitos, diga quantos possuem exatamente três dígitos 9 e os
quatro dígitos restantes todos diferentes?

Temos um total de P73 .C9, 4 números de sete dígitos com três repetidos, veja figura abaixo.

9 9 9

P73 C 9, 4

Mas, devemos excluir os números iniciados em zero, que são em um total de P63 .C8,3

Então teremos P73 .C9, 4 - P63 .C8,3 = 126 x 840 – 56 x 120 = 99120 números.
7. De quantas maneiras podemos distribuir 22 livros diferentes entre 5 alunos se 2 deles recebem 5
livros cada e os outros 3 recebem 4 livros cada?

Primeiro escolhemos os alunos de C5, 2 . C3,3 maneiras diferentes.

Depois escolhemos os livros de C22,5 . C17,5 . C12, 4 . C8, 4 . C4, 4 maneiras diferentes.

O total, pelo principio multiplicativo, será o produto das duas escolhas.

22!
Teremos C5, 2 . C3,3 . C22,5 . C17,5 . C12, 4 . C8, 4 . C4, 4 = = 56463835428000
5! . 4! . 3! . 2!
3

8. Quantos são os números naturais de sete dígitos nos quais o dígito 4 figura exatamente 3 vezes e o
dígito 8 figura exatamente 2 vezes?

Os números iniciados em 4 são iguais a 1. C6, 2 . C4, 2 . 8 . 8 = 5760 ( C 6, 2 representa as maneiras de


acrescentar os outros dois dígitos 4 e C 4, 2 representa as maneiras de acrescentar os dois dígitos 8).

Os números iniciados em 8 são iguais a 1. C6, 1 . C5,3 . 8 . 8 = 3840 ( C 6, 1 representa as maneiras de


acrescentar o outro dígito 8 e C 5, 3 representa as maneiras de acrescentar os três dígitos 4).

Os números iniciados em outros dígitos, exceto o zero, são iguais a 7 . C6, 3 . C3, 2 . 8 = 3360 ( C 6, 3
representa as maneiras de acrescentar os três dígitos 4 e C3, 2 representa as maneiras de acrescentar
os dois dígitos 8).

Portanto, pelo princípio aditivo, temos a soma 5760 + 3840 + 3360 = 12960 números.

9. De quantas maneiras uma comissão de 4 pessoas pode ser formada, de um grupo de 6 homens e 6
mulheres, se a mesma é composta de um número maior de homens do que de mulheres?

Se o número de homens é maior que o número de mulheres então devemos ter 3 homens e 1 mulher
ou 4 homens e 0 mulheres.

Para 3 homens e 1 mulher temos C6,3 . C6,1  120 maneiras de escolher as comissões.

Para 4 homens e 0 mulheres temos C6, 4  15 maneiras de escolher as comissões.

Pelo principio aditivo temos 120 + 15 = 135 maneiras de escolher as comissões.


10. O comprimento de uma palavra é a quantidade de caracteres que ela possui. Encontre a
quantidade de palavras de comprimento 5 que podemos formar fazendo uso de 10 caracteres distintos,
de forma que não existam três caracteres consecutivos idênticos em cada palavra.

Usando os 10 caracteres temos 10 5 maneiras de formar as palavras de comprimento 5.

São indesejáveis as palavras cujo formato está mostrada na figura abaixo.

A A A B C

Grupo único

O grupo que repete pode ser contado como 10 . 3! (são 10 caracteres diferentes e temos três itens
para permutar AAA, B e C.

Para as duas posições que sobram podemos ter 9 . 8 = 72 maneiras de escolha (pois esses caracteres
não podem ser iguais).

Portanto temos 10 5 – 10 . 3! . 9 . 8 = 95680 palavras.

11. Quantos números inteiros existem entre 1 e 10.000 que não são divisíveis por 3; 5 e 7?

Obs.: Fiz o exercício sem considerar que fossem múltiplos simultâneos.

São 3333 múltiplos de 3, 1999 múltiplos de 5 e 1428 múltiplos de 7.

São 666 múltiplos de [3, 5] = 15; 476 múltiplos de [3, 7] = 21 e 285 múltiplos de [5, 7] = 35.

São 95 múltiplos de [3, 5, 7] = 105.

Pelo princípio da inclusão e exclusão temos 10000 – [3333 + 1999 + 1428 – (666 + 476 + 285) + 105] =
4562 números que não são múltiplos de 3, 5 e 7.
12. Quantas são as permutações da palavra PROPOR nas quais não existem letras consecutivas
iguais?

O método usado foi através da fixação de uma das letras e determinando as possibilidades de
posicionar as demais letras sem haver repetição.

Cada linha da tabela abaixo mostra as posições escolhidas para uma letra.

P 2 P 1 1 1 Fixando o P dessa forma temos 2 . 1 . 1 . 1 = 2 possibilidades


P 2 1 P 2 1
P 2 1 1 P 1 Fixando o P dessa forma temos 2 . 1 . 2 . 1 = 4 possibilidades
P 2 1 1 1 P Fixando o P dessa forma temos 2 . 1 . 1 . 1 = 2 possibilidades

Fixando o P dessa forma temos 2 . 1 . 1 . 1 = 2 possibilidades

Temos um total de 2 + 4 + 2 + 2 = 10 possibilidades usando a letra P.

Como são três letras, então temos 3 x 10 = 30 palavras nas quais não existem letras consecutivas
iguais.

13. De quantos modos 6 casais podem sentar-se ao redor de uma mesa circular de tal forma que
marido e mulher não fiquem juntos?

Primeiro posicionamos o grupo de mulheres que pode ser feito de 5! = 120 maneiras.

Em seguida devemos posicionar os homens observando a restrição imposta.

Designando os casais como (M1, H1); ... ; (M6, H6) sabemos que o Hi não pode ficar ao lado de Mi.

Na figura abaixo verificamos que existem duas posições proibidas para cada Hi.

Proibido para H1

Proibido para H1

Então podemos posicionar os homens de 4! = 24 maneiras distintas.

Logo, pelo principio multiplicativo, temos 24 x 120 = 2880 maneiras de posicionar os seis casais
obedecendo à restrição imposta.
14. Quantas são as permutações das letras da palavra BRASIL em que o B ocupa o primeiro lugar, ou
o R ocupa o segundo lugar, ou o L o sexto lugar?

No diagrama ao lado temos:


B R

4!
Anagramas em que o B ocupa o
primeiro lugar 5!.
5! 5!

4! 3! 4!
Anagramas em que o R ocupa o
segundo lugar 5!.

5!

L Anagramas em que o L ocupa o sexto


lugar 5!.

Anagramas contados com repetição de B e R 4!.

Anagramas contados com repetição de B e L 4!.

Anagramas contados com repetição de L e R 4!.

Anagramas com as três condições do problema 3!.

Portanto, pelo princípio da inclusão e exclusão, temos um total de 5! + 5! + 5! – (4! + 4! + 4!) + 3! = 294
anagramas que contemplam o enunciado.

15. De quantas formas podemos representar o número 15 como soma de vários números naturais?

Não encontramos um padrão de combinatória para a solução.

Foi resolvido, em sala, relacionando caso a caso e encontrou-se 176 formas.

16. Quantos quadrados perfeitos existem entre 40.000 e 640.000 que são múltiplos simultaneamente
de 3, 4 e 5?

Os múltiplos simultâneos de 3, 4 e 5 são múltiplos de [3, 4 , 5] = 60 = 22 . 3 . 5. Seja M um número


natural múltiplo de 60 e que seja quadrado perfeito temos então que M = 60.Q = 2 2.3.5. Q , onde
Q  N de forma que tenhamos, no mínimo, Q = 3.5.7 2 e no máximo Q = 2 2.3.5.132 , pois pela
condição do problema M deve ser maior que 40000 e 60 . 3.5.7 2 = 2 2.3.5.3.5.7 2 = 2 2.32.5 2.7 2 = 44100
e 60 . 2 2.3.5.132 = 2 2.3.5. 2 2. 3. 5.132 = 608400 que representam, respectivamente, o primeiro
quadrado perfeito múltiplo de 60 maior que 40000 e o maior quadrado perfeito múltiplo de 60 menor
que 640000.
Fazendo uma análise dos números quadrados perfeitos que cumprem o enunciado percebemos que
eles são da forma P 2 . 900 , como 44100  49 . 900  7 2. 900 e 608400  676 . 900  26 2. 900 então
temos que 7  P  26 . Portanto são 26 – 7 + 1 = 20 números quadrados perfeitos que cumprem as
condições do enunciado.

17. Oito amigos vão ao cinema assistir a um filme que custa um real. Quatro deles possuem uma nota
de um real e quatro possuem uma nota de dois reais. Sabendo-se que o caixa do cinema não possui
nenhum dinheiro, como eles podem organizar uma fila para pagar o filme permitindo o troco pelo
caixa?

De acordo com a árvore de possibilidades acima temos 14 caminhos diferentes para o posicionamento
dos oito amigos.

O grupo que possui um real cada pode ser posicionado de 4! = 24 maneiras na fila, da mesma forma
os que possuem dois reais cada também podem ser posicionados de 4! = 24 maneiras na fila.

Então, pelo principio multiplicativo, temos um total de 14 . 24. 24 = 8064 maneiras de dispor os amigos
na fila.

18. Se considerarmos todas as configurações do tabuleiro com duas torres que não se atacam, como
no Exemplo 5.2, sem distinguir as torres, quantas configurações obteremos?

No exemplo 5.2 as torres são diferentes e representam 64 . 7 2  3136 maneiras distintas para
posicioná-las de forma que não se ataquem. Para o caso proposto as torres são idênticas portanto
64 . 7 2
temos  1568 configurações.
2!
19. Continuando o problema anterior, generalize-o para 3, 4, 5, ... torres que não se atacam,
encontrando também o número máximo de torres que podem ser colocadas no tabuleiro de modo que

duas delas não se ataquem.

Observando a solução do problema 18 e a figura ao lado


temos que 64 representa o número de casas para colocar a
primeira torre, 72 representa o número de casas que a
segunda torre pode ocupar (posicionando a primeira sobra
uma quantidade de casas equivalente à um quadrado com 7
casas de lado) e o 2 ! corrige o fato das torres serem iguais
(a permutação não gera uma nova disposição).

82 . 7 2 . 6 2
Para três torres temos configurações, para
3!
8 2 . 7 2. 6 2. 5 2
quatro torres temos e para n torres temos
4!
8 2. 7 2... 8  n   1
2

n!

O máximo acontece para n = 8. Basta posicionar todas as torres na diagonal.

20. Tente fazer o problema anterior para cavalos de xadrez.

Iniciei com as figuras abaixo, mas não consegui concluir.

Cavalo na segunda ou Cavalo na 3a ou 4a ou 5a ou


Cavalo nos vértices. sétima coluna das laterais. 6a coluna das laterais.

Cavalo 1  4 opções Cavalo 1  8 opções Cavalo 1  16 opções


Cavalo 2  61 opções Cavalo 2  60 opções Cavalo 2  59 opções
21. Mostre que em toda sequência de n2 +1 inteiros distintos possui uma subsequência crescente de n +
1 elementos ou uma subsequência decrescente de n + 1 elementos.

Seja um inteiro positivo e suponhamos, por contradição, que haja uma sequência de 2+1 inteiros
distintos que não contenha uma subsequência monótona (cresceste ou decrescente) de tamanho +1,
ou seja, todas as subsequências monótonas de tem tamanho , no máximo. Seja um elemento da
sequência . Associamos a um par ordenado de inteiros , , onde é o comprimento da maior
subsequência crescente de que começa em e é o comprimento da maior subsequência
decrescente de que começa em . Sobre as subsequências monótonas de podemos afirmar que:

𝑖) o máximo de pares ordenados de inteiros utilizados é 2. De fato, como o comprimento de uma


subsequência monótona de é no máximo igual a , utilizamos inteiros de 1 a . Logo, utilizamos no
máximo . = 2 pares ordenados de inteiros;

𝑖𝑖) dois elementos distintos de não podem estar associados ao mesmo par ordenado de inteiros. De
fato, suponhamos que e são elementos de S, com ≠ . Seus pares ordenados são , e , e
como ≠ , temos que < ou > . Se < , afirmamos que > ·, pois sabemos que existe uma
subsequência crescente de comprimento começando em . Se inserirmos no começo dessa
subsequência, obtemos uma subsequência crescente de comprimento +1. Assim, ≥ +1 ou,
equivalentemente, > . Assim e têm pares ordenados diferentes. Da mesma forma, se > , então
> e, novamente e estão associados a pares ordenados diferentes.

Entretanto, essas duas afirmações levam a uma contradição. Há apenas 2 pares ordenados de inteiros
diferentes, e tem 2+1 elementos. Pelo PCP (Principio da Casa dos Pombos), dois dos elementos
devem ter o mesmo par ordenado. Entretanto, isto contradiz o fato de que dois elementos quaisquer
não podem estar associados ao mesmo par ordenado. Portanto deve ter uma subsequência
monótona de comprimento +1.

22. Encontre o número de zeros que termina o número 2010!.

Usando a técnica de contar os grupos das potências de 5 (que multiplicados pelas potências de 2
resulta em zeros) temos:

Temos 2010 : 5 = 402 grupos de 51.

Temos 402 : 5 = 80 grupos de 52.

Temos 80 : 5 = 16 grupos de 53.

Temos 16 : 5 = 3 grupos de 54.

Como 54 é a maior potência de 5 presente em 2010, temos um total de 402 + 80 + 16 + 3 = 503 zeros
na terminação de 2010!.
23. O jogo do 7 consiste em lançar dois dados e somar o número obtido nas suas faces. Caso a soma
seja 7, o jogador A ganha dois reais do jogador B. Caso a soma não seja 7, o jogador B ganha um real
de A. Pergunta-se: quem leva vantagem?

Quem leva vantagem é o jogador B, pois no lançamento de dois dados temos 6 2  36 somas
diferentes. Dentre estas apenas 6 dão total sete. Portanto em cada jogada temos uma probabilidade de
1 5
a favor do jogador A e a favor do jogador B.
6 6

24. A função  de Euler associa a cada número natural n o valor (n) igual ao número de inteiros
positivos menores ou iguais a n relativamente primos com n. Ou seja,  (n)  1  m  n ; m, n  1

Usando os princípios estudados, mostre que se n se decompõe

  k  1  1   1 
em fatores primos como n  p1 1 . p2 2 ... pk , então  (n)  n 1   . 1   ... 1  .
 p1   p2   p k 

 1  1   1   p  1   p2  1   pk  1 
 (n)  n 1   . 1   ... 1    n. 1 . ... 
 p1   p2   pk   1  2   k 
p p p

 p1  1. p2  1... pk  1   k
 (n)  n. , substituindo n  p1 1 . p2 2 ... pk temos:
p1 . p 2 ... p k

 p1  1. p2  1... pk  1
 (n)  p1 . p 2  ... p k  .
1 2 k
dividindo as potências dos fatores primos de n resulta
p1 . p 2 ... p k

 (n)  p1 1 . p2 1 ... pk  1 . p1  1


1 2 k
. p2  1... pk  1
Capítulo 6 - Soluções dos Exercícios

QUESTÃO 1
n(n  1 )( 2.n  1)
Se qn denota a soma qn = 12 + 22 + ... + n2, prove que para todo n N qn 
6
Vamos provar do indução finita.
1 - A afirmação é verdadeir a para n  1, pois :
1( 1  1 )( 2.1  1)
qn  12  1 qn  1
6
2 - Hipótese de indução, que é verdadeir a para n  k com k  N , então :
k ( k  1)( 2k  2)
12  2 2  ...  k 2 
6
Tese de indução, para n  k  1, então :
( k  1)( k  2)( 2k  3)
12  2 2  ...  k 2  ( k  1) 2 
6
Demonstração :
k(k  1 )( 2k  2 )
Somando ( k  1) 2 nos dois membros de 12  2 2  ...  k 2  teremos :
6
k(k  1 )( 2k  1 )
12  2 2  ...  k 2  ( k  1) 2   ( k  1) 2
6
k(k  1 )( 2k  1 )  6( k  1) 2
12  2 2  ...  k 2  ( k  1) 2 
6
k(k  1 )( 2k  2 )  6( k  1) 2
( k  1)k ( 2k  1)  6( k  1) ( k  1)( 2k 2  7k  6)
  
6 6 6
3
( k  1)2( k  2)( k  )
2  ( k  1)( k  2)( 2k  3)
6 6
Logo,
( k  1)( k  2)( 2k  3)
12  2 2  ...  k 2  ( k  1) 2 
6

QUESTÃO 2
Use o princípio da indução para provar as seguintes afirmações:
(a) 32n+1 + 2n+2 é divisível por 7 para todo n N;
Vamos provar do indução finita.
1 - A afirmação é verdadeir a para n  1, pois :
32.11  21 2  33  23  35 que é divisível por 7.
2 - Hipótese de indução, que é verdadeir a para n  k com k  N , então :
7q  32 k 1
32 k 1  2 k  2 é divisível por 7, então 32 k 1  2 k  2  7q, logo 2 k 
4
Tese de indução, para n  k  1, então :
32 k  3  2 k  3 é divisível por 7
Demonstração :
Desenvolvendo a Tese, teremos
32 k  3  2 k  3  27.32 k  8.2 k , substituindo a hipótese teremos :
7q  3.32 k
32 k  3  2 k  3  27.32 k  8( )  27.32 k  14q  6.32 k  7(3.32 k  2q)
4
Que é divisível por 7.
(b) a soma dos cubos de três números naturais consecutivos é divisível por 9;
Demonstrar que n2 + (n+1)2 + (n+2)2 é divisível por 9.
Vamos provar do indução finita.
1 - A afirmação é verdadeir a para n  1, pois :
13  23  33  36 é divisível por 9.
2 - Hipótese de indução, que é verdadeir a para n  k com k  N , então :
n 3  ( n  1) 3  ( n  2) 3 é divisível por 9, então n 3  ( n  1) 3  ( n  2) 3  9q
Tese de indução, para n  k  1, então :
( n  1) 3  ( n  2) 3  ( n  3) 3 é divisível por 9
Demonstração :
Isolando ( n  2) 3 na hipótese teremos,
( n  2) 3  9q  n 3  ( n  1) 3
substituindo ( n  2) 3 na tese, concluimos que :
( n  1) 3  ( n  2) 3  ( n  3) 3  ( n  1) 3  9q  n 3  ( n  1) 3  ( n  3) 3
( n  1) 3  ( n  2) 3  ( n  3) 3  9q  n 3  n 3  9n 2  27n  27
( n  1) 3  ( n  2) 3  ( n  3) 3  9( q  3n 2  9n  9)
Que é divisível por 9.

Vamos provar do indução finita.


1 - A afirmação é verdadeira para n  1, pois:
(1011  9.1  10) 81
7  7.  7.  7
81 81
2 - Hipótese de indução, que é verdadeira para n  k com k  N , então:
(10k 1  9k  10)
7  77  777  ...  777...7  7.
k  vezes 81
Tese de indução, para n  k  1, então:
(10k  2  9( k  1)  10) (10 k  2  9k  19)
7  77  777  ...  777...7  7.  7.
k 1 vezes 81 81
Demonstração:
Somando 777...7 nos dois membros da hipótese teremos
k 1 vezes

(10k 1  9k  10)
7  77  777  ...  777...7  777...7  7.  777...7
k  vezes k 1 vezes 81 k 1vezes
k 1
(10  9k  10) (10 k 1  9k  10)
7.  777...7  7.  7.111...1
81 k 1 vezes 81 k 1 vezes

x n 1  1
7.111...1  7.(10k  ...  102  10  1), lembrando que  x n  ...  x 2  x  1, então
k 1 vezes x 1
10k 1  1 10k 1  1 9 9.10 k 1  9
7.(10k  ...  102  10  1)  7.  7. .  7.
10  1 10  1 9 81
(10k 1  9k  10) (10 k 1  9k  10)
7.  7.111...1  7.  7.(10 k  ...  10 2  10  1) 
81 k 1 vezes 81
(10k 1  9k  10) 9.10 k 1  9 (10.10 k 1  9k  10  9) (10 k  2  9k  19)
7.  7.  7.  7.
81 81 81 81
Vamos provar do indução finita.
1 - A afirmação é verdadeira para n  1, pois:
(n  1)(n  2)(n  3)...(n  n)  2n.1.3.5...(2n  1)
(1  1)  2  21.1.(2.1  1)  2.1  2
2 - Hipótese de indução, que consideraremos verdadeira para n  k com k  N , então:
(k  1)(k  2)(k  3)...(k  k )  2k .1.3.5...(2k  1)
Tese de indução, para n  k  1, então:
(k  1  1)(k  1  2)(k  1  3)...(k  1  k  1)  2k 1.1.3.5...2(k  1)
(k  2)(k  3)(k  4)...(k  k )(2k  1)(2k  2)  2k 1.1.3.5...(2k  1)
Demonstração:
Vamos trabalhar com o primeiro mebro da tese
(k  2)(k  3)(k  4)...(k  k )(2k  1)(2k  2) 
(k  2)(k  3)(k  4)...(k  k )(2k  1).2(k  1)  (k  1)(k  2)(k  3)(k  4)...(k  k ).2(2k  1) 
hipótese de indução

Substituindo a hipótese teremos :


2k .1.3.5...(2k  1).2(2k  1)  2k 1.1.3.5...(2k  1).(2k  1) . Temos a validade de nossa tese.

QUESTÃO 3
Use o princípio da indução para provar as seguintes desigualdades:

A desigualdade 2  n n

a  b  a  b não é válida para todo n natural, pois ela não é válida para
n1 n

n  1. Para n  1, ela fica a  b  a  b , que não é verdadeiro. Mas, de fato, a desigualdade


 
2 n1 a n  b n  a  bn é válida para todo n  2 . Ela é válida para n  2 , pois para n  2 , ela fica
 
2 a 2  b 2  a  b2 , que equivale a 2a 2  2b 2  a 2  b 2  2ab 2 , que equivale a a 2  b 2  2ab  0 ,
que equivale a a  b   0 , que é verdadeiro, já que a  b . Suponha que a igualdade vale para n e
2

vamos mostrar que ela vale para n  1 , isto é, vamos mostrar que 2 a
n
 n1

 b n1  a  bn1 , isto é,
vamos mostrar que a  b
n1
 
 2 n a n1  b n1 . De fato, como a  bn1  a  bn a  b , a  b  0 e,
por hipótese de indução, a  b  2
n n1
a n
 
 b n , então a  bn1  2 n1 a n  b n a  b . Assim, para 
concluir que a  b
n1
 
 2 n a n1  b n1 é suficiente mostrar que 2 n1 a n  b n a  b  2 n a n1  b n1 .    
Mas, 2 n1
a n
b n
a  b  2 a n n1
b n1
 equivale a 2a n1
b n1
  a n
 b a  b , que equivale a
n

2a  2b  a  b  a b  ab , que equivale a a n1  b n1  a n b  ab n , que equivale a


n1 n1 n1 n1 n n

 
a n a  b n b  a n b  ab n , que equivale a a n  b n a  b  0 , que é verdadeiro, pois se a  b , então
a  b  0 e a  b  0 , sendo portanto a
n n n
b n
a  b  0 , e se a  b , então a n
 bn  0 e a  b  0 ,
sendo portanto a  b a  b  0 .n n
1 1 1
A desigualdade    n não é válida para todo n natural, pois ela não é válida para
1 2 n
1
n  1. Para n  1, ela fica  1 , que não é verdadeiro. Mas, de fato, a desigualdade
1
1 1 1
   n é válida para todo n  2 . Ela é válida para n  2 , pois para n  2 , ela fica
1 2 n
1 1 2 1
  2 , que equivale a  2 , que equivale a 2  1  2 , que é verdadeiro. Suponha
1 2 2
que a igualdade vale para n e vamos mostrar que ela vale para n  1 , isto é, vamos mostrar que
1 1 1 1
    n 1. De fato, como, por hipótese de indução,
1 2 n n 1
1 1 1 1 1 1 1 1
   n , então     n . Assim, para concluir que
1 2 n 1 2 n n 1 n 1
1 1 1 1 1
    n 1 é suficiente mostrar que n  n 1 . Mas,
1 2 n n 1 n 1
nn  1  1
nn  1  1  n  1 , que
1
n  n  1 equivale a  n  1 , que equivale a
n 1 n 1
equivale a nn  1  n , que equivale a nn  1  n 2 , que equivale a n  0 , o que é verdadeiro, pois
n2.

1 1 1 13
A desigualdade    não é válida para todo n natural, pois ela não é válida para
n 1 n  2 2n 24
1 13
n  1. Para n  1, ela fica  , que não é verdadeiro. Mas, de fato, a desigualdade
2 23
1 1 1 13
   é válida para todo n  2 . Ela é válida para n  2 , pois para n  2 , ela fica
n 1 n  2 2n 24
1 1 13 7 13
  , que equivale a  , que é verdadeiro. Suponha que a igualdade vale para n e vamos
3 4 24 12 24
mostrar que ela vale para n  1, isto é, vamos mostrar que
1 1 1 1 1 13
     . De fato, como, por hipótese de indução,
n2 n3 2n 2n  1 2n  2 24
1 1 1 13 1 1 13 1 1 1 13 1
   , então    . Como    ,
n 1 n  2 2n 24 n2 2n 24 n  1 n2 2n 24 n  1
1 1 1 1 1 13 1 1 1
então         . Assim, para concluir que
n2 n3 2n 2n  1 2n  2 24 n  1 2n  1 2n  2
1 1 1 1 1 13
     é suficiente mostrar que
n2 n3 2n 2n  1 2n  2 24
13 1 1 1 13 13 1 1 1 13
    . Mas,     equivale a
24 n  1 2n  1 2n  2 24 24 n  1 2n  1 2n  2 24
1 1 1 1 4n  3
  , que equivale a  , que equivale a
n  1 2n  1 2n  2 n  1 2n  12n  2
2n  12n  2  n  14n  3 , que equivale a 4n 2  6n  2  4n 2  7n  3 , que equivale a n  1  0 , o
que é verdadeiro.

QUESTÃO 4

Mostre a seguinte identidade trigonométrica

(n  1) cos nx  n cos(n  1) x  1
cos x  2cos 2 x  ....  n cos nx 
2 x
4sen 2
x
 2(1  cos x) , logo nossa identidade a ser demonstrada pode
2
Vamos considerar a identidade 4sen 2
ser reescrita com o

(n  1) cos nx  n cos(n  1) x  1
P(n): cos x  2cos 2 x  ....  n cos nx 
2(1  cos x)

Vamos demonstrar P(n) utilizando indução finita.

Vamos inicialmente testar n = 1, então

P(1)  cos x 
2cos 2 x  co s(2) x  1 2cos x  2cos

2
x 
2cos x(1cos x)  cos x (V)
2(1  cos x) 2(1  cos x) 2(1  cos x)

Hipótese: Vamos supor que P(n) é verdadeira para n = k

(k  1) cos kx  k cos(k  1) x  1
P(k )  cos x  2cos 2 x  ....  k cos kx 
2(1  cos x)

Tese: Vamos demonstrar que P(n) é verdadeira para n=k+1.

(k  2) cos(k  1) x  (k  1) cos(k  2) x  1
P(k  1)  cos x  2cos 2 x  ....  k cos kx  (k  1) cos(k  1) x 
2(1  cos x)

Somando (k+1)cos(k+1)x aos dois membros da hipótese teremos:

(k  1) cos kx  k cos(k  1) x  1
cos x  2 cos 2 x  ....  k cos kx  (k  1) cos(k  1) x   (k  1) cos(k  1) x 
2(1  cos x)
(k  1) cos kx  k cos(k  1) x  1  2(k  1) cos(k  1) x  2 cos x(k  1) cos(k  1) x

2(1  cos x)
cos(k  1) x  k  2k  2  2(k  1) cos x   (k  1) cos kx  1 cos(k  1) x  k  2  2(k  1) cos x   (k  1) cos kx  1
 
2(1  cos x) 2(1  cos x)
(k  2) cos(k  1) x  (k  1)  2 cos x cos(k  1) x  cos kx   1

2(1  cos x)
(I )

(k  2) cos(k  1) x  (k  1) cos(k  2) x  (k  1) (2 cos x cos(k  1) x  cos kx  cos(k  2) x )  1



2(1  cos x)
Vamos provar que (I) = 0.
Primeiramente observe que:
cos(k+2)x=cos[(k+1)x+x]=cos(k+1)x.cosx-sen(k+1)x.senx (II)
e
coskx=cos[(k+1)x-x]=cos(k+1)x.cosx+sen(k+1)x.senx (III)
Então trabalhando com (I) teremos que
2 cos x cos(k  1) x  cos kx  cos(k  2) x 
2 cos x cos(k  1) x  (cos(k+1)x.cosx+sen(k+1)x.senx+cos(k+1)x.cosx-sen(k+1)x.senx)=
2 cos x cos(k  1) x  2 cos x cos(k  1) x  0
Com isso provamos que a identidade trigonométrica é verdadeira.

QUESTÃO 5

Um torneio de xadrez tem n jogadores. Cada jogador joga uma única partida com cada um dos outros
jogadores. Calcule o número total de partidas realizadas no torneio.

n n!
Cn   2   2! n  2 ! . Observe que
2
O número de jogos realizados no torneiro será dado pela são
 
n jogadores, a partida é realizada entre dois jogadores e cada dupla joga uma única vez.
Vamos considerar P(n) como sendo o total de jogos realizados, ou seja que
n!
P(n) = . Vamos demonstrar a validade de P(n) para todo n natural maior que 1, utilizando
2! n  2 !
indução finita.

Para n = 2 é fácil verificar que será uma única partida.

2!
P(2)  1
2! 2  2 !

Como hipótese vamos considerar que P(n) é válida para n = k, com k natural e maior que 2.

k! k.(k-1)
P( k )  
2! k  2 ! 2

Tese: Vamos demonstrar que para n = k+1, P(n) também é verdadeira.

(k  1)! (k+1).k
P(k  1)  
2! k  1! 2
Para demonstrar a validade de P(k+1) devemos perceber inicialmente que se para k jogadores o
k .(k  1)
número de jogos é dado por P(k )  (hipótese de indução), ao aumentarmos um jogador ,
2
teremos k jogos a mais pois, esse novo jogador irá jogar com os k outros jogadores.
Somando k à hipótese teremos:
k.(k 1) k  k  2k
2
k k
2
k (k 1)
P(k )  k   
2 2 2 2
Temos então a validade de nossa tese.
QUESTÃO 6

Vamos provar do indução finita.


1 - A afirmação é verdadeir a para n  1, pois :
1.(1  1) 
2

13    12  1
 2 
2 - Hipótese de indução, que é verdadeir a para n  k com k  N , então :
 k ( k  1) 
2

13  2 3  33  ...  k 3  
 2 
Tese de indução, para n  k  1, então :
 ( k  1)( k  2) 
2

13  2 3  33  ...  ( k  1) 3   
 2
Demonstração :
Somando (k  1 )2 nos dois membros da hipótese temos :
 k ( k  1) 
2

13  2 3  33  ...  k 3  ( k  1) 3     ( k  1) 3
 2 
 k ( k  1) 
2
 
k 2 ( k  1) 2  4( k  1) 3 ( k  1) 2 k 2  4k  4 ( k  1) 2 ( k  2) 2
 2   ( k  1)    
3

4 4 4
 ( k  1)( k  2) 
2

 2 
Logo.
 ( k  1)( k  2) 
2

13  2 3  33  ...  ( k  1) 3   
 2

QUESTÃO 7

Vamos provar do indução finita.


1 - A afirmação é verdadeir a para n  1, pois :
1
 1
a1  1    2  3
 1
2 - Hipótese de indução, que é verdadeir a para n  k com k  N , então :
k k
 1  1
a k  1    3  1    q  3
 k  k
Tese de indução, para n  k  1, então :
n 1
 1 
an 1  1   3
 n  1
Demonstração :
Pelo desenvolvimento do binômio de Newton temos :
n 1
 1  k 1
 k  1 1 1 k 1
 k  1 1
an 1  1        1  ( k  1).    
 n  1  ( k  1) k  1 n 2  n  ( k  1)
n n
n 0  n
k 1 k 1
1 1 1
 1  1   k  1   k  3  k 1  3
n 2 2 n 2 2
QUESTÃO 8

Vamos provar do indução finita.


1 - A afirmação é verdadeir a para n  1, pois :
1  4a  1
a
2
2 - Hipótese de indução, que é verdadeir a para n  k com k  N , então :
1  4a  1
a  a  a  ...  a 
  2
k  radicais

Tese de indução, para n  k  1, então :


1  4a  1
a  a  a  ...  a 
  2
k 1 radicais

Demonstração :
Sendo a  0, então vamos somar a aos dois membros da hipótese :
1  4a  1
a  a  a  a  ...  a  a 
  2
k  radicais

1  4a  1
a  a  a  a  ...  a  a 
  2
k  radicais
 
k 1 radicais

1  2a  4a  1
a  a  a  a  ...  a 
 2
k 1 radicais

1  2a  4a  1 1  4a  1
Sendo,  , logo :
2 2
1  4a  1
a  a  a  a  ...  a 
 2
k 1 radicais
QUESTÃO 9

Vamos provar do indução finita.


1 - A afirmação é verdadeir a para n  1, pois :
1112  12 2.11  113  12 3  23.133 que é divisível por 133.
2 - Hipótese de indução, que é verdadeir a para n  k com k  N , então :
133q  11k 2
11k 2  12 2 k 1 é divisível por 133, então 11k 2  12 2 k 1  133q, logo 12 2 k 
12
Tese de indução, para n  k  1, então :
11k 3  12 2 k 3 é divisível por 133
Demonstração :
Desenvolvendo a Tese, teremos
11k 3  12 2 k 3  1331.11k  1728.12 2 k , substituindo a hipótese teremos :
 133q  11k 2 
11k 3  12 2 k 3  1331.11k  1728.   113.11k  133.1728q  144.112.11k
 12 
k 3 2 k 3
11  12  133(1728q  11 .11 )  133(1728q  11k 2 )
2 k

Que é divisível por 133.

QUESTÃO 10

Vamos provar do indução finita.


1 - A afirmação é verdadeir a para n  1, pois :
32.11  212  33  23  35 que é divisível por 7.
2 - Hipótese de indução, que é verdadeir a para n  k com k  Z  , então :
7q  32 k 1
32 k 1  2 k 2 é divisível por 7, então 32 k 1  2 k 2  7q, logo 2 k 
4
Tese de indução, para n  k  1, então :
32 k 3  2 k 3 é divisível por 7
Demonstração :
Desenvolvendo a Tese, teremos
32 k 3  2 k 3  27.32 k  8.2 k , substituindo a hipótese teremos :
2 k 3 k 3 7q  3.32 k
3  2  27.3  8( 2k
)  27.32 k  14q  6.32 k  7(3.32 k  2q)
4
Que é divisível por 7.
Questão 11

Mostre que para todo n  2


+ 2n+2 6 n1
Z temos que 3 é um múltiplo de 11.

Vamos fazer a demonstração por indução finita.

Consideremos n = 0.
6.0 1
2
2.0+2 2 1
Substituindo teremos 3 = 3 +2 =11 que é múltiplo de 11.

Hipótese de Indução - Suponhamos que seja verdade para n=k , com k  +


Z , temos que
6.k 1
2 = 11q com q 
2.k+2 +
3 Z
Tese
6.k  7
2
2.k+4
Vamos demonstrar que para n = k+1 , 11 divide 3
6.k  7
2 .3  2 .2 .
2.k+4 2k+2 2 6k 7
Desenvolvendo nossa tese teremos 3 =3

Agora iremos trabalhar com a hipótese.


6.k 1
3
2.k+2
2 = 11q  3
2.k+2
 11q  26.k 1( I )
Substituindo (I) na tese teremos que

(11q 26k 1 ).3  2 .2  9.11q 9. 2  2 .2  9.11q 18.2 128 .2  9.11q 110.2  11(9q 10. 2 )
2 6k 7 6 k 1 6k 7 6k 6k 6k 6k

Como 11(9q10. 26k ) é divisível por 11, provamos que a tese também é verdadeira.

Logo provamos que 11 divide 32n+2  26n1 para todo n  Z


+
.

Questão 12
n n
   
1 . 1 5  1 . 1 5 
Considere Fn a sequência de Fibonacci. Mostre que Fn  
5  2 
 5  2 

Um solução:

A sequência de Fibonacci corresponde à recorrência F n


 F n1  F n2, com F F
1 2
 1 . Temos
então que essa sequência é 1,1,2,3,5,8,13,21,34,.......
n n
   
Vamos demonstrar a validade de  1 . 1 5   1 . 1 5  , que define cada termo dessa
Fn
5  2 
 5  2 

sequência. Vamos utilizar indução finita.

Para n = 1
1 1

1 . 1 5  1  1 5   
Temos que
F    .   1 . 1 5 1 5 
 1
5  2  2
1 
 5   5  2 

Como hipótese, consideremos que F n


é válido para n = k.

k k

n = k temos que
F  1 . 1 5 
  1 . 1 5 

5  2 5  2
k

 
Nossa tese será provar que para n = k+1 , F n
também é válida.

Para n = k+1 ,
k 1 k 1
 1 5  1 . 1 5  
F  1 .  
5  2  5  2 
k 1

k k

1 . 1 5  . 1 5  1  1 5   1 5
  .

 2   2  .
.
5  2   2  5   
Temos que F k 1
 F k  F k 1 ,

Somando
k k k 1 k 1
 1 5  1  1 5  1  1 5   1 5 
F  F  1 .   .   .   1 .  
5  2  5  2  5  2 
k k 1
5  2 
k k k 1 k 1

1 . 1 5   1 . 1 5   1 . 1 5  . 1 5   1 . 1 5  . 1 5 


 
5  2  5  2  5  2   2  5  2   2 
k k k k

1 . 1 5   1 . 1 5   1 . 1 5  . 2   1 . 1 5  . 2  


   
5  2  5  2  5  2   1 5  5  2   1 5 
k k

1 . 1 5   1 . 1 5  
 
 
 . 1 2  . 1  2
5  2 5  2
 
  1 5    1 5 

k k

1  1 5   3 5 
 1  1 5   3 5 
.  .  .  .  
 2   2 
  1 5
   1 5 
5   5    
k k

1 . 1 5   1 5 1 . 1 5   1 5 


 .   .  
 F
5  2 5  2
 k 1
   
  2    2 

Outra solução
n
Consideremos as progressões geométricas v n  q , com q  0 , que satisfaz à recorrência
x x
n n 1
 x n2, com x x
1 2
1
.

Temos que
n n 1 n2
q q q
cujas soluções são
´ 1 5 ´´ 1 5
q 2
eq 
2
´n ´´m
q w q
Defina v n
e n
.

Como v ew
n n
satisfazem à recorrência x x
n n 1
 x n2, com x x1 2
1
,

então, para todo  e  reais, a sequência Fn   v w


n n
também satisfaz à recorrência.
Agora impomos F1  F 2  1, o que nos dá um sistema de duas equações com as duas
1 1
incógnitas  e  , cujas soluções são  = e  =- .
5 5

Questão 13

Mostre as seguintes propriedades a respeito da sequência de Fibonacci F n


:

n n
a) F  F
i 1
i

n2 1
b) F i 1
2 i 1
 F 2n

F  F n  (1)
n
 F 2 n1 1
2
c)
i 1
2i
d) F .F n 1 n 1

Resolução.
n
a) F  F
i 1
i n2
1

Para n = 1 temos que F  F 1 1


1 3
Hipótese: Vamos supor que seja verdade para n = k com k natural e maior que 1
F F 1 2
 ............  F k  F k 2 1

Tese: Vamos demonstrar que a fórmula é verdadeira para n = k+1

F F 1 2
 ............  F k  F k 1  F k 3 1
Substituindo a hipótese no primeiro membro da igualdade temos que:
F 1  F
k+2 k 1
 F k 3 1 , o que é verdade pois, F k 3
 F k 1  Fk+2
n
b) F  F
i 1
2 i 1 2n

Para n = 1 temos:
F  F 1
1 2

Verdadeiro
Como hipótese vamos supor a validade da fórmula para n = k, sendo k natural maior que 1
F F 1 3
 .............  F 2k 1  F 2 k
Tese: Vamos demonstrar que a fórmula é verdadeira para n = k+1.
Para n = k+1 temos
F F 1 3
 .............  F 2k 1  F 2k 1  F 2k  2
Considerando a tese temos que
F F 1 3
 .............  F 2 k 1  F 2 k 1  F 2 k  2
hipótese

Substituindo a hipótese, então


F 2k
 F 2 k 1  F 2 k  2

O que mostra a validade da Tese.

c)
n

F
i 1
2i
 F 2 n1 1

Vamos demonstar a validade desta propriedade utilizando indução finita.


Para n = 1 é válida a propriedade.
F 2
 F 3 1  2 1  1
Como hipótese de indução vamos considerar que essa propriedade é válida para n = k , com k
natural e maior que 1.
Para n = k teremos:
F F 2 4
 .........  F 2k  F 2 k 1  1
Tese : Vamos demonstrar que a propriedade é válida para n = k+1
Para n = k+1
F F 2 4
 .........  F 2k  F 2k 2  F 2k 3  1
Trabalhando com a tese

F F 2 4
 .........  F 2 k  F 2 k  2  F 2 k 3  1
hipótese de indução

Substituindo a hipótese de indução


F 2 k 1
1  F 2k 2  F 2k 3  1  F 2 k 1  F 2 k 2  F 2 k 3
O que demonstra a validade de nossa tese.

d)

F n1.F n1  F n  (1)


2 n

Para n= 2 temos a validade da propriedade.

F .F  F 1.212  1 (1) .
2 2
1 3 2

Hipótese: Vamos supor que a propriedade é válida para n = k com k natural e maior que 2.
Para n = k temos
 F k  (1)
2 k
F .F k 1 k 1

Tese: Vamos demonstrar que a propriedade também é válida para n = k+1.


Para n = k+1 temos que:
 F k 1  (1)
2 k 1
F .F k k 2

Para demonstrar a validade da tese, vamos inicialmente trabalhar com a hipótese:

 F k  (1)
2 k
F .F k 1 k 1
Multiplicando os dois membros por (-1) obtemos

 F k 1.F k 1  F k  (1)
k 1

2

 F k 1 F k 1  (1) k 1


2

 F k 1.F k 1  

 F k 1.F k 1  F k 1  2F k 1.F k 1  F k 1  (1)


k 1

2 2

 3F k 1.F k 1  F k 1  (1)
2 2 k 1
F k 1

Agora iremos trabalhar com o primeiro membro da tese


F .F  F   F  F  .  2 F  F   F 
2 2
k k 2 k 1 k 1 k 1 k 1 k 1 k 1

2 F  F .F  2 F F  F  F 
2 2 2
k 1 k 1 k 1 k 1 k 1 k 1 k 1

2F  3F F  F  F  F  F  3F F  F k 1  F k 1
2 2 2 2 2 2 2
k 1 k 1 k 1 k 1 k 1 k 1 k 1 k 1 k 1
hipótese de indução

Substituindo a hipótese de indução na tese, obtemos:


 (1)  F k 1  (1)
2 k 1 2 k 1
F k 1

Com isso demonstramos a validade da nossa tese.

Questão 14

De quantas formas diferentes podemos cobrir um tabuleiro de 2 x n com peças de dominós que cobrem
exatamente duas celas do tabuleiro?

Considere um tabuleiro com 2 linhas e n+2 colunas. Para preencher o canto esquerdo do tabuleiro, há
duas alternativas: colocar um dominó “em pé” , restando um tabuleiro com 2 linhas e n+1 colunas a
preencher( iremos preencher o que falta de xn+1 maneiras) ou colocar dois dominós “deitados” restando
um tabuleiro com 2 linhas e n colunas (que poderá ser preenchido de xn maneiras distintas). Logo, o
número xn de modos de preencher um tabuleiro 2 x n com dominós 2 x 1 satisfaz a recorrência
xn+2= xn+ xn+1, com x1 = 1 e x2 = 2. Esta é a sequência de Fibonacci, logo temos
n 1 n 1

1  1 5  1  1 5 
x n
 F n
     
5 2  5 2 
Questão 15

. Calcular o número de regiões em que o plano é dividido por n retas distintas em cada uma das
seguintes situações:

(a) as n retas são concorrentes;


(b) não existem duas retas paralelas nem três retas concorrentes

a) As n retas concorrentes dividem o plano em 2n regiões.


Vamos demonstrar tal fato por indução finita.

Para n = 1, ou seja uma reta, o plano fica dividido em 2.1 = 2 regiões.

Vamos supor que seja verdade que para n = k o plano fica dividido em 2k regiões ( Hipótese de
indução)

Vamos verificar que para n = k+1 o plano fica dividido em 2(k+1) regiões.
Temos que em um conjunto de k retas, cada reta acrescentada (deve ser uma reta concorrente
às outras retas) aumenta mais duas regiões. Ou seja para k retas temos 2k regiões,
acrescentando mais uma reta teremos 2k + 2 regiões e 2k+2=2(k+1) que demonstra nossa tese.

c)Fazendo alguns desenhos que satisfazem ao problema verificamos que:

Número de regiões
Número de retas (n)
P(n)

1 2

2 4

3 7

4 11

5 16

O número de regiões P(n) em que o plano fica dividido por n retas, sendo duas não paralelas e
n(n+1)
nem três concorrentes é dado por P(n)= 1.
2

Vamos demonstrar a validade de P(n) utilizando indução finita.

Inicialmente vamos considerar n = 1. Vemos facilmente que para uma reta o plano fica dividido
em duas regiões.

1.(1+1)
P(1)= 1  2
2

k.(k+1)
Hipótese: Vamos considerar que para n = k seja validade P(k) = P(k)= 1 .
2
Tese: Vamos demonstrar que para n = k+1 seja válida P(k+1).

(k+1).(k+2)
Considerando que para k retas o plano fica dividido em P(k)=  1 . Ao acrescentarmos mais
2
uma reta ela acrescenta k+1 novas regiões. Observe que a nova reta ao encontrar a primeira reta, ela
separa em duas regiões a região em que está, entrando em outra região. Ao encontrar a segunda reta,
ela separa em duas regiões a região em que está, entrando em outra reta, e assim sucessivamente até
encontrar a n-ésima reta. Assim são obtidas k+1 regiões a mais das que já existiam, logo:

k.(k+1) 
2

 1  k+1= k k  k 11 k
2
3k  2  (k 1).(k  2) . C.q.d
P(k+1) = P(k) + k+1 =
2
1 1
2 2 2
Questão 16

Dizemos que uma figura é enquadrável com régua e compasso, se a partir dela é possível, utilizando
apenas régua e compasso, construir um quadrado de mesma área. Prove que:

(a) um triângulo é sempre enquadrável;


(b) um polígono qualquer é enquadrável.

Sugestão para o item (b): Utilize indução dividindo a figura em triângulos.

a+b+c
Seja um triângulo de lados a,b e c. Temos que A= p(p-a).(p-b).(p-c) sendo p= .
2
Seja x = p.(p-a).(p-b).(p-c)

h
L

X 1 h 1

Temos que h2= x . 1 Podemos escrever L2= h . 1

Podemos construir um quadrado com lado L = h que terá área igual a A= p(p-a).(p-b).(p-c) .

b) Considere P(n): Todo polígono de n lados é enquadrável.

Base de indução.

P(3) é verdadeira pelo item anterior.

Suponhamos que P(k) é válida.

Supondo P(k) verdadeira, vamos provar que isto implica em P(k+1) verdadeira.

Todo polígono com (k+1) lados, ao traçarmos uma diagonal “conveniente”, é particionado em um
triângulo e um polígono com k lados. Sabemos construir um quadrado Q1 que tem área igual a de um
triângulo e sabemos construir Q2, um quadrado com área igual a área de um polígono de k lados (pela
hipótese de indução). Seja f o lado de Q1 e g o lado do quadrado Q2. Temos que f2 é a área do quadrado
Q1 e g2 é a área do quadrado Q2. Pelo teorema de Pitágoras temos que f 2 + g2 = m2 , onde m2 é a área
de um novo quadrado com lado m.

Questão 17

Dê uma resposta à situação á Observação 6.17.

Sugestão: Observe a validade do argumento quando o conjunto A tem 2 elementos. Veja que B e C não
se intersectam. Ou seja, o passo indutivo falha de n = 1 para n = 2.

O erro se encontra no passo indutivo de n=1 para n=2.

Para n=1 é verdade, pois se há uma camisa todas tem a mesma cor.
Agora vamos considerar n = 2, ou seja há duas camisas. Não posso afirmar que as duas tem a mesma
cor, o que foi considerado como verdadeiro na demonstração na observação 6.17.
Vamos tentar demonstrar que se n = 1 é verdadeiro então n =2 também é verdadeiro.

Para n = 2 .
Consideremos o conjunto B = {b1, b2}
Vamos dividir em dois subconjuntos
B´ = { b1} e B” ={ b2} , temos que B = B´ U B”
é fato que não podemos garantir que
B´ ∩ B” ≠ 
Capítulo 7 – Soluções dos Exercícios

1 – Provar que em todo triângulo a soma dos comprimentos das medianas é menor que o perímetro do
triângulo e maior que o semiperímetro deste.
Observe a figura, onde o triângulo ABC tem lados a, b e c e as medianas tem medidas m a,
(mediana relativa ao lado a ) ,m b (mediana relativa ao lado b) e mc (mediana relativa ao lado c)

Vamos provar inicialmente que a soma dos comprimentos das medianas de um triângulo é maior que o
semiperímetro deste, ou seja:
abc
m m m
a b c

2
Temos
c
mc  2  b
b
mb  2  a
a
mc  2  b
Somando as desigualdades obtemos:
abc c.q.d
m m m
a b c

2

Agora iremos provar que em um triângulo a soma dos comprimentos das medianas é menor que
o perímetro do triângulo. Prolonguemos, por exemplo a mediana mb e a partir do vértice A traçamos
um segmento de comprimento a e paralelo ao lado BC que intersectará o prolongamento de mb em D.
Faremos processos análogos às medianas mc e ma

Temos que:
2m b  a  c
2m a  b  c Somando as desigualdades teremos: m m m
a b c
 a  b  c c.q.d
2m c  a  b
2 –Os centros de três círculos que não se intersectam estão sobre uma reta. Prove que se um quarto
círculo toca de forma tangente os três círculos, então o raio deste é maior que pelo menos um dos raios
dos três círculos dados.

O caso em que uma das três circunferências é tangente internamente à circunferência de raio R já
satisfaz a condição de R ser maior que o raio de pelo menos uma das circunferências.

Vamos analisar agora outra situação.

Considere a figura em que as três circunferências com centros sobre uma reta t, são exteriores duas a
duas e tangentes externas à quarta circunferência. Seja C1 de raio R1, C2 de raio R2 e C3 de raio R3.
A quarta circunferência tem raio R e centro C. Considere x o espaçamento entre as circunferências 3 e
2 e y, o espaçamento entre as circunferências 2 e 1

Pela figura, considerando o triângulo de lados R + R3 , R + R1 e R1+x+2R2+y+R3, podemos escrever:

R  2 R  R  x  y  2R  R  R
1 2 3 1 3

2 R  x  y  2R ,
2

2 R 2 R
2

R R 2

CQD.

n
1 2n
3 – Dado n inteiro positivo, provar que  j  n 1
j 1

Vamos utilizar o fato que a média aritmética de um conjunto de números reais é maior que ou igual a
média harmônica desses valores.
MA  MH
1 1 1 1
  .......  , multiplicando a desigualdade por n, escrevendo 1+2+ ....+ n como
2 n
n 1  2  ....  n
n
n(n+1)/2, obtemos:
n
1 1 1 2n 1 2n
  .......  
2 n n 1
que podemos escrever como  j  n 1
j 1
c.q.d

4 – A soma de três números positivos é 6. Provar que a soma deseus quadrados não é menor que 12.

Sejam a, b e c números reais positivos.


Temos que a + b + c = 6.
Vamos utilizar o fato de que
MA  MQ

a b c
2 2 2
abc
 
3 3

a b c
2 2 2

2 
3
 
2 2 2

4 a b c 
3
12  a  b  c
2 2 2

5 – Determinar as dimensões do paralelepípedo de menor diagonal possível, sabendo que a soma dos
comprimentos de todas suas arestas é 12.

Considere um paralelepípedo retângulo de dimensões a, b e c, como o comprimento de todas suas


arestas é 12, podemos escrever, 4a + 4b + 4c = 12, então a+ b+ c = 3.

a b c
2 2 2
Seja d a diagonal, sabendo que d = e usando o fato de que MA≤MQ, teremos que:

a b c
2 2 2
abc
 
3 3

a b c
2 2 2

1 
3
 
2 2 2

1 a b c 
3
3 a b c
2 2 2

Como queremos a menor diagonal, tomaremos d = 3 e sabemos que a igualdade MA = MQ, ocorre
quando a=b=c e neste caso teremos a = b= c = 1.

6 – Encontrar todas as soluções positivas do sistema de equações não lineares

Considerando a primeira equação do sistema podemos escrever que a média aritmética desses valores
é:
  .............. x10
2 2 2

MA  x x
1
1 2
 e utilizando a segunda equação do sistema, podemos escrever
10 10
que
10 10 1
MH    .
1 1 1 100 10
2
 2
 ..........  2
x x 1 2 x 10

Temos que:
MA  MH  x1  x 2  ......  x10
2 2 2

 1  x1  1/ 10 , mas queremos soluções


2
Substituindo na primeira equação, obtermos 10 x
1
positivas, logo
1
x x1 2
 ......  x10 
10

7 – Demonstrar que, se a , a ,....., a


1 2 n
são números positivos tais que
1
a .a ......a
1 2 n
Então (1+a1) . (1+a2) ......... (1+an )  2n.

Suponhamos por absurdo que (1+a1) . (1+a2) . (1+an) < 2n. Dividindo por 2n, obtemos:
(1  a1) (1  a 2) (1  a n)
. ........... 1 .
2 2 2
Iremos usar o fato de que MG≤MA. Observe que para todos os ai´s, com i {1, 2,.....n} , teremos que
1 ai
 1.a i  a i
, logo:
2
n
a .a ...........a
1 2 n
 1,
Sabemos que
a .a ...........a
1 2 n
= 1 (hipótese do problema),
Teríamos o fato de 1< 1 , o que é um absurdo. Logo vale a desigualdade
(1+a1) . (1+a2) ......... (1+an )  2n.

8 – Prove que a média geométrica é super-aditiva, isto é, para números não negativos
a ebi i
, 1  i  n, tem-se
n n n
n
 a i  n  bi  n  (a  bi)
i 1 i 1 i 1
i

Além disso, estude em que condições ocorre a igualdade.


Sugestão: Use a desigualdade entre as médias geométrica e aritmética.

Temos que MA≥MG, logo


 a
n

 a
n
i i
n
a b
i 1 i i

a b
i 1 i i
(I)
n

 b
n

 b
n
i i
n
a b
i 1 i i

a b
i 1 i i
(II)
n

Fazendo (I)( II )

 a  b
n n

 a   b
n n
i i n
i
 n
i

a b
i 1 a b i i i 1 i i

a bi 1 i i a bi 1 i i

n

 a b
n n n

a b
i 1
i i
n
 a i  n  bi
i 1 i 1
i i
 
n n
n
 (a  b )
i 1
i i

n n n n
n
 a i  n  bi n
 a i  n  bi
n  i 1 i 1
1 i 1 i 1
 n
n

 (a  b )   a n
n n
 n  b i c.q.d
i i
n n
n

 (a  b ) n
n

 (a  b )
i 1
i
i 1 i 1
i i i i
i 1 i 1

9 - Usar o método de indução para provar a desigualdade de Cauchy-Schwarz.

a1b1  a 2b2  ...........  a nbn  a1  a 2  ........  a n . b1  b2  ........  b n


2 2 2 2 2 2
Considere P(n) =

Vamos demonstrar que P(n) é válida para todo n natural, e para tal utilizaremos a indução finita.

Para n = 1 verificamos que P(n) é válida. Observe :


2
a1b1  a1 b1  ( a1b1)
2 2

Hipótese de indução: Suponhamos que P(n) seja verdadeira para n = k, então teremos
P(k)= a1b1  a 2b2  ...........  a kbk  a1  a 2  ........  a k . b1  b 2  ........  b k
2 2 2 2 2 2

Tese:

Vamos provar que para n=k+1 , P(n) é verdadeira.

Sendo n = k+1, temos que

a1b1  a 2b 2  ...........  a kb k  a k 1b k 1  a1b1  a 2b 2  ...........  a kb k  a k 1b k 1 


a1  a 2  ...........  a k . b1  b 2  ...........  b k  a k 1. b k 1  a1  a 2  .....  a k 1. b1  b 2  .....  b k 1.
2 2 2 2 2 2 2 2 2 2 2 2 2 2

Vamos verificar a validade desta última desigualdade.

Considere x, y, z e w números reais positivos. Utilizando o fato de que MG≤MA podemos escrever:
xw  yz
xw. yz   2 xw. yz  xw  yz  ( xy  zw  2 xw. yz  xy  xw  yz  zw 
2
2 2 2
( xy)  2 xw. yz  ( zw)  ( y  w).( x  z )  ( xy  zw )  ( y  w).( x  z ) 

x . y  z . w  x  y . y  w.

x a1  a 2  ...........  a k , y = b1  b2  ...........  bk , z = a k+1 e w= bk+1


2 2 2 2 2 2 2 2
Basta considerarmos

e a desigualdade fica demonstrada.

 (ai bi)
n 2
10 – Para todo λ real 0
i 1
Use este fato para dar outraprova da desigualdade de Cauchy-Schwarz

Podemos escrever:

 (ai bi)
n n n n n
  (a  2.a ib i.  b i . )   a i  2.  a i.b i   b
2 2
0
2 2 2 2 2
i i
i 1 i 1 i 1 i 1 i 1
Fazendo:
n n n
v   bi ,  a i.b i e t = a
2 2
u= i
,
i 1 i 1 i 1

Obtemos a desigualdade do segundo grau em .


t  2u  v 0
2

Temos que v > 0 e 0


  4u  4vt 0
2

4u  4vt 0
2

4u  4vt
2

Então teremos que

u  v.t
2

u  v.t
n n n
Que é a desigualdade de Cauchy-Schwarz, com v  b  a .b a
2 2
i
, u= i i
et= i
,
i 1 i 1 i 1

11 – Use a desigualdade de Cauchy-Schwarz para dar uma prova alternativa da desigualdade entre as
médias aritmética e quadrática (ma ≤ mq).

n n n

a b   a . b b  1, a
2 2
A desigualdade de Cauchy-Schwarz nos diz que i i i i
, fazendo i
i 1 i 1 i 1

n n

a   a .n
2
desigualdade poderá se reescrita como i i
que equivale a
i 1 i 1
2
a a  ..........  a n  (a  a  .........  a n ). n ,
2 2
1 2 1 2

Dividindo por n, os dois membros da desigualdade já que n é um número natural e positivo, podemos
escrever:
a a  .........  a n a a  ........  a n
2 2 2
1 2
 1 2
, logo MA  MQ . cqd
n n

n
1 n n

12 – Prove que  a b  2  ai2  bi2 
i 1
i i
i 1 i 1

Uma solução
n
1 n 2 n 2 
Queremos provar que 
i 1
a ibi  2 
i 1
a i 
i 1
bi 
n n

1 n 2 n 2   a 2 
i  b 2
i
Temos que  
2  i 1 a i i 1
bi  
i 1 i 1

2
, que consideraremos a média aritmética de dois

termos.
n n n

a b   a i . bi ,
2 2
A desigualdade de Cauchy-Schwarz nos diz que i i
i 1 i 1 i 1

Usando o fato de que MG  MA


n n

n n n
1 n 2 n 2   a 2 
i  b 2
i
 a i.bi   a i . bi    bi  
2 2 i 1 i 1

i 1 i 1 i 1 2  i 1 a i i 1 2

13 – Prove que a4 + b4 + c4 ≥ abc(a + b + c)

Uma solução , considerando a, b e c reais.

Inicialmente vamos considerar o fato de que MQ≥MA

a b c  a b  c
2 2 2

3 3
a  b  c  (a b  c)
2
2 2 2

 3.(a  b  c )  (a b  c)
2 2 2 2
Podemos escrever
3 9 ,

Desenvolvendo esta última desigualdade concluiremos que a2 + b2 + c2≥ ab+ ac + bc .

A partir desta conclusão podemos escrever a4+b4+c4=(a2)2 + (b2)2+(c2)2≥ a2 b2+ a2 c2+ b2 c2 (I)

Temos o fato de que a2 b2+ a2 c2+ b2 c2 ≥ (ab+ac+bc)2/3. (II)

Para justificarmos a desigualdade II basta observarmos que

(ab ac bc) (ab ac)  2.(ab ac).bcb2c 2 a b  2a bc  a 2c 2 2ab c  2abc  b c


2 2 2 2 2 2 2 2 2

  =
3 3 3
a b  a b  b c  . a bc  ab c  abc ≤ 2 2  2 2  2 2 .
2 2 2 2 2 2 2 2 2

2 ab ab bc
3 3
Vamos trabalhar com a desigualdade II, temos que

a2 b2+ a2 c2+ b2 c2≥(ab+ac+bc)2/3 que equivale a

a b  a b  b c  . a bc  ab c  abc ≥ a 2bc  ab2c  abc 2  abc(abc) ( III )


2 2 2 2 2 2 2 2 2

a b ≥
2 2 2 2 2 2
ab c c 2 .
3 3
Pela desigualdade I temos que

a b c  a b  a c b c
4 4 4 2 2 2 2 2 2

e a desigualdade III nos fornece que

a b  a c  b c  abc(abc)
2 2 2 2 2 2

Logo, concluímos que

a4 + b4 + c4 ≥ abc(a + b + c)

Outra solução considerando a, b e c ≥ 0

Inicialmente vamos considerar o fato de que MA≥MG


a  b  a 4.b 4  a 4  b 4  a 2.b 2
4 4

2
2
a  c  a 4.c 4  a 4  c 4  a 2.c 2
4 4

2
2
b  c  b 4.c 4  b 4  c 4  b 2.c 2
4 4

2
2
Somando estas desigualdades
2a  2b  2c  2a b  2a c  2b c 
4 4 4 2 2 2 2 2 2

a  b  c  a b  a c  b c (I )
4 4 4 2 2 2 2 2 2

Utilizando novamente a desigualdade entre as médias aritmética e geométrica podemos escrever:


a b  b c  a 2b 2.b 2c 2  a 2b 2  b 2c 2  2a b 2.c
2 2 2 2

2
a b  a c  a 2b 2.a 2c 2  a 2b 2  a 2c 2  2a 2b .c
2 2 2 2

2
a c  b c  a 2c 2.b 2c 2  a 2c 2  b 2c 2  2a b .c 2
2 2 2 2

Somando as desigualdades obtemos


2a b  2a c  2b c  2a bc  2ab c  2abc 
2 2 2 2 2 2 2 2 2

a b  a c  b c  a bc  ab c  abc 
2 2 2 2 2 2 2 2 2

a b  a c  b c  abc(a b c) ( II )
2 2 2 2 2 2
A desigualdade I nos diz que

a b c  a b  a c b c
4 4 4 2 2 2 2 2 2

e a desigualdade II nos fornece o fato de que


a b  a c  b c  abc(a bc)
2 2 2 2 2 2

Comparando I e II concluímos que

a  b  c  abc(a bc)
4 4 4

c.q.d

14 – Prova que se a≥0, b≥0 e c ≥ 0. Então (a + b)(a + c)(b + c) ≥ 8abc:

Vamos usar o fato de que MA  MG

ac
 a.c  a  c  2. ac
2
bc
 bc  b  c  bc
2
Multiplicando as inequações, obtemos:
(a+b) . (a+c) . (b+c) ≥ 8.a.b.c c.q.d

15 – Queremos provar que (1+x)n> 1+nx para todo x positivo e n natural ,

Uma solução
Temos que (1+x)n> 1+nx  (1  x)  n 1  nx
Sendo n natural e maior que um.
Considere os números positivos a1, a2, ......an, tais que

a1. a2. ...... .an = 1 + nx (produto dos números)


e
a1+ a2+ .....+.an = n + nx
Podemos dizer que um dos ai, com i  {1,2,...n} é igual a 1+nx. Vamos supor que a1 = 1 + nx, logo
teremos que a2=a3=........=an = 1. Usando o fato de que MG≤MA,

nx  n
 n 1  nx  x  1  1  nx  (1 x) , como a ocorre para a1=a2=.....=an
n
n
1  nx 
n
E nossos valores não serão todos iguais a um teremos o que queríamos demonstrar

(1+x)n> 1+nx

Outra solução
Agora iremos demonstrar tal desigualdade utilizando indução.

(1+x)n> 1+nx
Observe que para n = 1 teremos (1+x)1 = 1 + 1.x, ou seja uma igualdade, logo não vale para n = 1.
Iremos fazer a nossa indução para todo n natural e maior que 1.

Então faremos o teste para n = 2


(1+x)2 > 1+2x , vamos inicialmente desenvolver (1+x)2 = 1 + 2x + x2, como x é positivo, então x2 > 0,
logo
1+2x + x2 > 1 + 2x, o que prova a desigualdade para n = 2.
Agora como hipótese, consideremos que para um n=k, com k natural e maior que 1, a desigualdade
(1+x)k > 1+kx seja verdadeira .

Vamos verificar se a hipótese verdadeira, implica que para n = k + 1 a desigualdade se torna verdadeira
(tese).
A nossa tese será então para n = k+1
(1+x)k+1 > 1+(k+1)x.

A partir da hipótese, podemos multiplicar os dois membros da desigualdade por (1+x), pois se x é
positivo, então 1+x também será positivo.
(1+x)k > 1+kx  (1+x)k+1> (1+kx) (1+x). Agora vamos trabalhar com (1+kx).(1+x) = 1+x + kx + kx2 =
1+(k+1)x + kx2,
como kx2 é positivo, temos que 1+(k+1)x + kx2 > 1+(k+1)x, logo (1+x)k+1 > 1+(k+1)x. c.q.d

16- Prove que se a; b; c e d são inteiros positivos, então:

Vamos utilizar o fato de que MH≤MA.


Sejam quatro números inteiros positivos a, b, c e d. Temos que:
abcd 4
MA  e MH  , logo
4 1 1 1 1
  
a b c d

4 abcd 1 1 1 1
  (a  b  c  d ).(    )  16
1 1 1 1 4 a b c d
  
a b c d

17 - Prova que se a≥0, b≥0 e c ≥ 0, então

Vamos usar o fato de que MA  MG


Consideremos a, b e c três números maiores que ou iguais a zero.
Temos que
ab  bc
  b ac
2

2 acb
ba  ac
  a bc
2

2 bca
ac  cb
  c ab
2

2 abc
Somando as desigualdades, obtemos
(ab  bc  ac)
2  a bc  b ac  c ab
2
 (ab  bc  ac)  a bc  b ac  c ab c.q.d
18 - Prove que se x ≥ 0, então 3x3 - 6x2 + 4 ≥ 0.
Sugestão: Use a desigualdade entre as médias aritmética e geométrica.

Da inequação 3x3 - 6x2 + 4 ≥ 0, podemos escrever 3x3+ 4 ≥ 6x2


Vamos utilizar o fato de que MA≥MG.
Considere os monômios 2x3, x3 e 4
Temos que

4
3

MA  3x e MG  8x  2 x
3 6 2

3
4
3
3x  2x 
2

Então, 3
3x  4 6x  3x  6 x  4  0
3 2 3 2

8
19 - Prove que se x≥ 0, então 2 x  4 x
3
Vamos utilizar o fato de que MQ≥MG.
Considere os monômios 3x e 4, temos que
2

MQ 
(3x)  4 2

2
e
então podemos escrever
MG  12 x
2
(3x)  4  2

2

2 12x  9x 2 16  12x  6x
Teremos então que
9 x 1612 x .(4) 
2

36 x  64 48 x 
2

36 x  48 x  640 
2

36 x  96 x 144 x  640 
2

36 x  96 x  64144 x 
2

(6 x 8)  144 x 
2

6 x 812 x : (3) 
8
2 x   4 x c.q.d
3
20 – Sejam C1 e C2 dois círculos concêntricos de raios r1 e r2, respectivamente, com r1 < r2. Sobre o
círculo C1 se marcam dois pontos P1 e P2 diametralmente opostos. Deseja-se encontrar o ponto P sobre
o círculo C2 que maximiza a soma d(P) = PP1 + PP2.

Pela desigualdade das médias temos que MA < MQ.

PP1  PP2 PP1 2  PP2 2



2 2

PP1 2  PP2 2
d ( P)  PP1  PP2  2 .
2

d ( P)  PP1  PP2  2 . PP1 2  PP2 2


d ( P) MAX  PP1  PP2 , ou seja, o triângulo P1 PP2 é isósceles.
Pela conclusão anterior temos que o triângulo POP1 é retângulo, portanto:

OP2  OP1 2  PP1 2 então r1  r2  PP1 


2 2 2

d ( P) MAX  2. r12  r2 2
Capítulo 8 – Soluções dos Exercícios

QUESTÃO 1
Calcule o quociente e o resto da divisão de p(x) por q(x) para os polinômios p(x) e q(x) dados:
(a) p(x) = 3x3 - 2x + 1 e q(x) = -7x - 1;
3x3 + 0x2 - 2x + 1 -7x-1
3 2
- 3x -3/7x -3/7x2 - 3/49x + 101/343 quociente
- 3/7x2 – 2x + 1
+3/7x2 - 3/49x
-101/49x+1
+101/49x -101/343
Resto -> 242/343

(b) p(x) = x5 - 1 e q(x) = x - 1;


x5 + 0x4 + 0x3 + 0x2 + 0x - 1 x - 1
- x5 + x4 x4 + x3 + x2 + x quociente
x4 + 0x3
- x4 + x3
x3 +0x2
- x3 + x2
x2 + 0x
- x2 + x
x-1
-x + 1
Resto 0

(c) p(x) = 3x5 - 2x3 + 1 e q(x) = x2 + x + 1


3x5 + 0x4 - 2x3 + 0x2 + 0x + 1 x2 + x + 1
- 3x5 - 3x4 – 3x3 3x3 - 3x2 - 2x + 5 quociente
4 3 2
- 3x - 5x + 0x
+ 3x4 + 3x3 + 3x2
- 2x3 + 3x2 + 0x
+ 2x3 + 2x2 + 2x
5x2 + 2x + 1
- 5x2 - 5x - 5
Resto - 2x - 4

QUESTÃO 2

Encontre os valores de A e B de forma que

Reduzindo a expressão ao mesmo denominador e igualando os coeficientes do 1º e 2º membros,


temos:
x 1 A B x  1 A( x  1)  B( x ) Ax  A  Bx x 1
   2    2
x  x x x 1
2
x x x x
2
x x
2
x x
A  B  1 A  B  1
   1  B  1  B  2
 A  1  A  1
Então A=-1 e B=2

QUESTÃO 3

Se os polinômios x2-x+4 e (x-a)2+(x+b) são iguais, encontre a + b.


Sendo os polinômios idênticos então iremos igualar os coeficientes do 1º e 2º membros de acordo com
os seus expoentes

x 2  x  4  x  a   x  b  x 2  x  4  x 2  2ax  a 2  x  b
2

x 2  x  4  x 2   2a  1x  a 2  b
 2a  1  1  2a  2  a  1
 2
a  b  4  1  b  4  b  3

Então a=1 e b=3

QUESTÃO 4

Quais os valores de a e b que tornam iguais os polinômios


P1(x) = x2 - x - 6 e P2(x) = (x + a)2 - b?

Sendo os polinômios idênticos então iremos igualar os coeficientes do 1º e 2º membros de acordo com
os seus expoentes:

P1  P2  x 2  x  6   x  a   b  x 2  x  6  x 2  2ax  a 2  b
2

 1
2a  1  a  2
 2
 
a  b  6     b  6  b  6  1  b  25
2 1
 2 4 4

QUESTÃO 5

A divisão de P(x) por x4 + 1 tem quociente x + 2 e resto 1. Encontre o polinômio P(x).


Usando o principio fundamental da divisão teremos:

P( x )  D( x )  Q ( x )  R( x )
P( x )  ( x 4  1)( x  2)  1
P( x )  x 5  2 x 4  x  2  1
P( x )  x 5  2 x 4  x  3

QUESTÃO 6

Qual o resto da divisão do polinômio x100 por x + 1?


De acordo com o teorema do resto temos que R = P(raiz do divisor), então:
R  P( 1)
R  ( 1)100
R 1

QUESTÃO 7

Determine o resto da divisão do polinômio p(x) pelo polinômio


g(x) = x, onde p(x) = (x - 1)(x - 2) ... (x - n) + b .
Usando o teorema do resto teremos:
R  P(0)
R  (0  1)(0  2)(0  3)...(0  n )  b
R  ( 1)( 2)( 3)...( n )  b
R  ( 1)n .n!b

QUESTÃO 8

Mostre que xn - 1 é divisível por x - 1 para todo n 1


Teremos que R= (1)n – 1 , como (1)n = 1 para n maior ou igual a 1, então R = 1 – 1, R= 0 logo xn - 1 é
divisível por x – 1.

QUESTÃO 9

Faça os seguintes itens:


(a) encontre o quociente da divisão de xn+1 - 1 por x - 1

Sabemos que x n 1  1  (ax n  bx n 1  ...  x )( x  1)  r ( x )


Pelo algoritmo de Euclides temos que P(x) = q(x ).d(x) + r (x).
Então q( x )  x n  x n 1  ...  x  1
(b) utilize a divisão anterior para calcular a soma 1 + x + x2 +x3 + ... + xn dos n primeiros termos de uma
progressão geométrica de razão x.
Usando a soma de termos de uma progressão geométrica teremos:

1  x  x  ...  x
2 n 1
x 
n 
1. ( x ) n 1  1 

x n 1  1
x 1 x 1
QUESTÃO 10

Determine o valor de a para que o polinômio P(x) seja divisível


por x - a, onde P(x) = x3 + (1 - a)x2 + (1 + a)x - 1

R  P( a )
R  a3  a2  a3  a  a2  1
R  2a 2  a  1
Sendo P(x) divisível por x-a então R = 0.
R0
2a 2  a  1  0
a1  1
a2  0

QUESTÃO 11

Mostre que o polinômio P(x) = x100 - 2x50 + 1 é divisível por x2 - 1.

Se P(x) for divisível por x2 – 1, então ele é divisível por x + 1 e x – 1, pois


x2 – 1 = (x + 1)(x – 1).
Então teremos: P(1) = 0 e P(-1) = 0

 P(1)  0  P( 1)  0
 100 
(1) 2(1)  1  0 ( 1) 2( 1)  1  0
50 100 50

 
1  2  1  0 1  2  1  0
verdade verdade

QUESTÃO 12

Mostre que o resto r(x) da divisão do polinômio p(x) por x - s é r(x) = p(s).

Na divisão do polinômio p(x) por x – s, teremos quociente de grau n-1, para p(x) de grau n e o resto terá
grau 0, logo o resto é uma constante que chamaremos de r.
Usando a equação fundamental da divisão teremos:

p( x)  ( x  s).q( x)  r

Tomando x = s, então:
p( s )  ( s  s ).q( s )  r
p( s )  0.q( s )  r
r  p( s )

Logo r = p(s) que é chamado de Teorema de D’Alambert.

QUESTÃO 13

Dado o polinômio p(x) = an xn + an-1xn-1 + ... + a1x + a0 definimos a derivada de p(x) como sendo o
polinômio, p'(x) = nanxn-1 + (n - 1)an-1xn-2 + ... + 2a2x + a1:
Por exemplo, a derivada do polinômio x5 é o polinômio 5x4 e a derivada do polinômio x3+5x2+2x-1 é o
polinômio 3x2+10x+2.
Usando as informações, calcule:

(a) a derivada dos polinômios:


(i) x + 1;

p' ( x )  1

(ii) x4 + 3;
p' ( x )  4 x 3
(ii) 1 + x + x2 + x3 + ... + xn.

p' ( x)  1  2 x  3x 2  ...  nx n 1

(b) Sabendo que p(0) = 1, calcule também o polinômio p(x) cuja derivada é
(i) x4.

1
p( x )  x 5  1
5
(ii) -x2 + 1.

1
p( x )   x 3  x  1
3
(ii) x3 + 2x2 + 3.

1 4 2 3
p( x )  x  x  3x  1
4 3
(c) Prove que se p(x) e q(x) são polinômios, então
(i) (p + q)’(x) = p’(x) + q’(x)
Dado p( x)  an x n  an 1 x n 1  ...  a2 x 2  a1 x  a0 e q( x)  bn x n  bn 1 x n 1  ...  b2 x 2  b1 x  b0
Derivando p(x) e q(x) teremos
p' ( x )  nan x n 1  (n  1)an 1 x n  2  ...  2a2 x  a1 x
q' ( x )  nbn x n 1  (n  1)bn 1 x n  2  ...  2b2 x  b1 x

Somando as derivadas:
p' ( x)  q' ( x)  n(an  bn ) x n 1  (n  1)(an 1  bn 1 ) x n  2  ...  2(a2  b2 ) x  (a1  b1 ) x
Somando os polinômios e derivando a soma dos polinômios temos:
( p  q)( x )  (an  bn ) x n  (an 1  bn 1 ) x n 1  ...  (a2  b2 ) x 2  (a1  b1 ) x  (a0  b0 )
( p  q)' ( x )  n(an  bn ) x n 1  (n  1)( an 1  bn 1 ) x n  2  ...  2(a2  b2 ) x1  (a1  b1 )
Logo teremos que:
( p  q)' ( x)  p' ( x)  q' ( x)

(ii) (pq)’(x) = p’(x)q(x) + p(x)q’(x)

QUESTÃO 14

QUESTÃO 15
(a) 1 + x2 + x4 + ... + x2n-2 é divisível por 1 + x + ... + xn-1?
Temos uma divisão de duas somas de PGs finitas, usando a fórmula de soma de PG finita e
simplificando elas teremos
1  x  x  ...  x
2 4 2n 2


1. ( x 2 ) n  1


( x n  1).( x n  1)
x2  1 ( x  1).( x  1)

1  x  x  ...  x
2
 n 1 
1. ( x ) n  1

xn  1 
x 1 x 1
Dividindo o primeiro polinômio pelo segundo teremos,
( x n  1).( x n  1)
( x  1).( x  1) ( x n  1).( x n  1) x  1 xn  1
 . 
xn  1 ( x  1).( x  1) x n  1 x 1
x 1
Para ser divisível temos n  2k  1

(b) 1 + x3 + x6 + ... + x3n-3 é divisível por 1 + x + ... + xn-1?


Temos uma divisão de duas somas de PGs finitas, usando a fórmula de soma de PG finita e
simplificando elas teremos

1  x  x  ...  x
3 6 3n  3


1. ( x 3 ) n  1


( x n  1).( x 2 n  x n  1)
x3 1 ( x  1).( x 2  x  1)

1  x  x  ...  x
2

1. ( x ) n  1
n 1

xn 1 
x 1 x 1
Dividindo o primeiro polinômio pelo segundo teremos,
( x n  1).( x 2 n  x n  1)
( x  1).( x 2  x  1) ( x n  1).( x 2 n  x n  1) x  1 x 2n  x  1
 . 
xn 1 ( x  1).( x 2  x  1) xn 1 x2  x 1
x 1
Para ser divisível temos n  3k  1
(c) Generalize.

Generaliza ndo, para p( x)  1  x p  x 2 p  ...  x pn p entao n  pk  1


QUESTÃO 16

(a) Resolva a equação 20x3 - 30x2 + 12x - 1 = 0, sabendo-se que1/2 é uma de suas raízes.
Usando o dispositivo de Briot-Ruffini, temos:
20 -30 12 1
1/2 20 -20 2 0
Então teremos a equação do 2º grau p( x)  20 x 2  20 x  2 com raízes
5  15 5  15
x1  ; x2 
10 10

(b) Uma raiz da equação x3 - (2a+1)x2 + a(a+2)x - a(a+1) = 0 é a + 1, ache as outras duas.

Usando o dispositivo de Briot-Ruffini, temos:


1 -(2a+1) a(a+2) -a(a+1)
a+1 1 -a a 0

Então teremos a equação do 2º grau p( x)  x 2  ax  a com raízes


a  a 2  4a a  a 2  4a
x1  ; x2 
2 2

QUESTÃO 17

Ache os possíveis valores de a Z para que o polinômio a2x4 + 4x3 + 4ax + 7


seja divisível por x + 1.
Sendo o polinômio divisível por x+1 então p(-1)=0, logo:
p( 1)  0
a 2 ( 1)4  4( 1)3  4a ( 1)  7  0
a 2  4a  3  0
a1  1
a2  3

QUESTÃO 18

Prove que todo polinômio de grau 1 é irredutível.

Seja f(x) um polinômio de grau 1.


Se f(x) fosse redutível em R, existiriam polinômios f(x), h(x) g(x)  R tais que
f(x) = h ( x ). g ( x ) com gr(g(x)) < gr(f(x)) = 1 e gr(h(x)) < gr(f(x)) = 1
Portanto, teríamos que gr(g(x)) = gr(h(x)) = 0, ou seja, g(x) e h(x) seriam polinômios
constantes, o que tornaria a igualdade f(x) = g(x )h(x) impossível, já que, de um lad,
temos um polinômio de grau um e, do outro, teríamos um polinômio de grau 0.
Assim concluimos que todo polinômio de grau um é irredutível.
QUESTÃO 19

Prove que se f(x) é um polinômio de grau maior ou igual a 2 e possui uma raiz real, então f(x) é
redutível.

Seja a é uma raiz de f(x), então f(a) = 0, logo f(x) é divisível por x - a .
Então f(x) se escreve da forma f(x) = (x - a).g(x), onde o grau de g(x)  1.
Portanto f(x) é redutível.
QUESTÃO 20

Mostre que todo polinômio f(x) de grau ímpar maior ou igual a 3 é redutível.

Se um certo número complexo é raiz de um polinômio com coeficientes reais, necessariamente seu
conjugado também será. Logo, havendo raízes complexas, estas serão em número par, existindo,
necessariamente, ao menos uma raiz real "a". Sendo, portanto, o polinômio divisível por x - a. Então
será redutível.
Obs. Se os coeficientes forem complexos, o Teorema Fundamental da Álgebra já garante a existência
de raízes, logo o polinômio será redutível.
QUESTÃO 21

(Critério de Eisenstein). Seja f(x) = a0 + a1x +...+anxn um polinômio com coeficientes inteiros. Suponha
que exista um primo p tal que:

Então, f(x) é irredutível sobre Q.

Mostre que os seguintes polinômios f(x) são irredutíveis sobre Q.


(a) f(x) = x4 + 2x3 + 2x2 + 2x + 2;
a) 2 não divide 1
b) 2 divide 2, 2, 2 e 2
c) 22 não divide 2
f(x) é irredutível sobre Q

(b) f(x) = x6 + 15;


a) 3 não divide 1
b) 3 divide 0 e 15
c) 32 não divide 15
f(x) é irredutível sobre Q

(c) f(x) = x4 + 10x3 + 20x2 + 30x + 22.


a) 2 não divide 1
b) 2 divide 10, 20, 30 e 22
c) 22 não divide 22
f(x) é irredutível sobre Q

QUESTÃO 22

Determine quais dos polinômios abaixo são irredutíveis sobre Q.


Sugestão: Use o critério de Eisenstein.
(a) x3 - x + 1
(b) x3 + 2x + 10
(c) x4 - x + 1
O único polinômio irredutível sobre Q é f(x) = x3 + 2x + 10, pois o número primo 2 não divide o
coeficiente an , divide os coeficientes a0, a1, ..., an-1 e 22 não divide o coeficiente a0.

QUESTÃO 23

QUESTÃO 24

Determine o polinômio p(x) de grau 7 tal que p(1) = p(2) = ...= p(7) = 8 e p(0) = 1:

O polinômio p( x ) de grau 7 deve ser da forma :


p(x) = A(x -1 )(x- 2 )...(x- 7 ) + 8
Como p(0)  1, então termos :
7 1
1  A(0  1)(0  2)...(0  7)  8  A   A
 7.6! 6!
1
Portando p( x )  ( x  1)( x  2)...( x  7)  8
6!

Você também pode gostar